Download as pdf or txt
Download as pdf or txt
You are on page 1of 156

434343

Question 1

An 18-year-old G1P0 woman presents in active labor at term gestational age.


A baby is born and the resuscitation is initiated, then an intubation is
attempted. The intubation attempt is unsuccessful likely because of the
infants physical findings of micrognathia, glossoptosis, and a U-shaped cleft
palate. Which of the following diagnosis most likely fit your findings?

A. DiGeorge syndrome
B. Edward syndrome
C. Pierre–Robin sequence
D. Treacher Collins syndrome
E. Goldenhar syndrome

Question 2

A 20-year-old female came to Dermato-venereology clinic because of


multiple flesh-colored papules around her genital area. The symptoms had
first appeared since 2 months. The patient had history having sexual
intercourse without using condom. Physical examination revealed
dome-shaped papules with central umbilication with caseous material.
Which of the following is a diagnosis of the patient?

A. Chancroid
B. Bartholinitis
C. Molluscum contagiousum
D. Genital herpes
E. Genital warts

Question 3
A 20-year-old woman, P0A0, came with a chief complaint of left quadrant
pain. She reports a sharp pain with associated discomfort most severe when
she lies on her left side that has progressed over the past 2-3 months. She is
sexually active with normal menstrual period cycle. Her last menstrual period
was 2 weeks ago. She denies any change in her bowel habits, urination or
weight. She has no significant past medical or gynecologic history of cancer.
The physical examination reveals a thin woman. Vital signs: BP 100/60 mmHg,
pulse 80/min and afebrile. Abdominal examination shows no ascites or
distention, there is slight tenderness to deep palpation in the left lower
quadrant without any masses or inguinal adenopathy and pelvic
examination reveals normal anatomy with a palpable, slightly tender, mobile
mass approximately 6 cm on bimanual examination on the left, with no
masses in the rectovaginal septum. A pelvic ultrasound performed 3 days
after her next menses which shows a complex left adnexal mass of 8 cm x 5
cm x 6 cm, a normal size uterus and right ovary, without any pelvic fluid noted
in the cul-de-sac. What is the most likely diagnosis for this patient?

A. Tubo-ovarian abscess
B. Ectopic pregnancy
C. Ovarian cyst
D. Endometrioma
E. Uterine fibroid

Question 4

A 25-year-old G1P0 patient at 41 weeks presents to labor and delivery


complaining of gross rupture of membranes and painful uterine contractions
every 2 to 3 minutes. On digital examination, her cervix is 3 cm dilated and
completely effaced with fetal feet palpable through the cervix. The estimated
weight of the fetus is about 2900 kg and the fetal heart rate tracing is
reactive. Which of the following is the best method to achieve delivery?
A. Deliver the fetus vaginally by breech extraction
B. Deliver the baby vaginally after external cephalic version
C. Perform a forceps-assisted vaginal delivery
D. Perform an internal podalic version
E. Perform an emergent cesarean section

Question 5

A 22-year-old G2P1AO patient was admitted into hospital at 32 weeks of


pregnancy with fluid leakage from her birth canal since 4 hours ago. A day
ago she has unprotected sexual intercourse with her husband. What is the
correct workup to diagnose the patient?

A. Litmus (Lakmus) test


B. Koch test
C. Cytology test
D. PCR test
E. complete blood count

Question 6

The investigators want to compare cholesterol level of the children whose


fathers have died from heart disease versus the children whose fathers do
not have a history of heart disease. Suppose from 100 children whose fathers
have died from heart disease, the observed mean and standard deviation of
the cholesterol level are 207.3 mg/dl and 35.6 mg/dl. Another 80 children
whose fathers do not have a history of heart disease are also identified and
the observed mean and standard deviation of the cholesterol level are 193.4
mg/dl and 17.3 mg/dl. It is assumed that the variances of the two groups are
not equal. If the assumptions for parametric test are not met, which of the
following is the correct alternative test?
A. Kolmogorov – Smirnov test
B. Kruskal – Wallis test
C. Wilcoxon signed-rank test
D. Wilcoxon sum-rank test
E. Friedman test

Question 7

A 54-year-old woman, P0A0, unmarried, came with a chief complaint of


vaginal bleeding 5 days ago. The patient needs to change pads every 4-5
hours because of the large amount of blood. The same complaints were also
felt in the last month, bleeding occurs for 20 days with a large amount of
blood. Complaints accompanied by a lump in the lower abdomen. She said
that she had not had her period in the previous 15 months. The patient
menarche when she was 10 years old. The patient’s mother died from colon
cancer. The patient had a history of uncontrolled type 2 diabetes mellitus
since 10 years ago. There is no history of blood clotting disorders. There is no
history of using estrogen replacement therapy. Patient nutritional status: BMI
= 28 kg/m2. On the abdomen, there is a palpable lump as big as a chicken
egg above the symphysis pubis with fixed, hard consistency, and tenderness
(-). Inspeculo examination showed that blood came out of the external
uterine ostium, the cervix was smooth. Bimanual examination revealed a
normal uterus with a slightly enlarged size. What is the most likely diagnosis
for this patient?

A. ;Cervical cancer knp ga ini?


B. Endometrial atrophy
C. Ovarian cyst
D. Endometrial cancer
E. Ovarian cancer
Question 8

A 23-year-old woman come with missed period for 5 weeks. She also feel
breast tenderness and the color become darker. She took home pregnancy
tests which all results positive. Which one below is the right answer?

A. miscarriage
B. probable pregnancy
C. positive pregnancy
D. presumptive pregnancy
E. suspect pregnancy

Question 9

A previously energetic woman complains of crying, loss of appetite, difficulty


in sleeping and feeling low self worth beginning 3 days after a normal vaginal
delivery. These feelings persisted for 1 week and progressively diminished.
Which is the following is the best term to describe her symptoms post
partum?

A. Schizoid affective disorders


B. Psychosis
C. Manic depression
D. Neurosis
E. Blues

Question 10

A newborn healthy baby girl, born at 36 weeks gestation. Her mother is a


27-year-old primigravid with no significant past medical history. The baby
delivered vaginally and was occurred for 10 hours. After delivery, there are
metabolic changes of the baby. From the following statements, which is the
correct one?

A. To maintain blood glucose after separation from the placental


circulation, the newborn experiences a deflate in catecholamine and
glucagon levels and an increase in insulin amounts.
B. Ketone bodies and lactate provide additional energy for the brain, with
hepatic ketogenesis deccreasing after the first 12 hours of age.
C. Cortisol levels begin to deflate at 30 weeks gestation and peak just
following delivery.
D. There is a progressive increase in metabolic rate which occurs more
slowly in preterm infants
E. Gluconeogenesis and glycogenolysis in the liver are increase until oral
intake volumes improve over the first few days after birth.

Question 11

A 19-year-old woman presented to the clinic with the primary complaint of


vaginal discharge that had been present since the previous week. She
confessed to being a promiscuous woman. The doctor found a frothy cream
discharge under the speculum and a motile protozoan with whip-like tail and
with jerky, twisting locomotion activity. What is the appropriate treatment for
such cases?

A. Amoxycylin 3x500mg for the patient


B. Metronidazole 4x250mg for the patient → pasangannya ganti2 soalnya
C. Metronidazole 3x250 mg for 7 days, given to the both partners
D. Cefixim 400mg per oral for the patient
E. Ceftriaxon 250 IM for that only patient

Question 12
What day of the ovarian cycle might you expect to see low levels of estrogren
and progresteron and high levels of FSH and LH ?

A. Day 21
B. Day 1
C. Day 14
D. Day 7
E. Day 28

Question 13

An 18-year-old patient finally delivered a 4,000-g infant vaginally. Her


prenatal course was complicated by anemia, poor weight gain, and maternal
obesity. She also complained that the production of her breastfeeding milk is
not much. On abdominal palpation, the uterine fundus is soft and at the level
of umbilicus. Wich of the following is correct about events that happen after
parturition?

A. Lactation results from stimulation of pituitary gonadotropin


B. The re epithelialized of endometrium will start after 1 week of parturition
C. In four weeks, the weight of the uterus will become as small as it was
before pregnancy
D. The uterus will involute 12 weeks after parturition
E. The content of the lochia, that is secreted for 10 days, is blood

Question 14

A 25-year-old patient complained about heavy menstrual bleeding and pain


in her abdomen. Her pain subsides if she is not in her menstruation period
Which of the following is not the proposed theories mechanism of the
disease?
A. Metaplasia of endometrial lining
B. Coelemic metaplasia
C. ;Ectopic transplantation
D. Induction theory
E. Immunologic

Question 15

A 65-year-old woman complained about cervical mass and the biopsy result
was cervical keratinizing squamous cell carcinoma. During the operation, the
tumor mass has extended to the pelvic wall. The staging for this patient is...

A. Stage IIB
B. Stage IV
C. Stage III
D. Stage I
E. Stage IIA

Question 16

Baby boy was born from G4P3 mother in 39 weeks of gestation based on first
day of last menstruation. At first minute and five minutes of life the doctor
examines five criteria: activity (tone), pulse, grimace, appearance, and
respiration. The result show that the baby in a good condition after birth. The
doctor plans to do gestational age measurement after birth. What methods
can be used?

A. Mentzer Score
B. APGAR Score
C. New Ballard score
D. Downes Score
E. Tollner score

Question 17

A 32-year-old pregnant woman is referred by her midwife for her high blood
pressure. She is in her second month of pregnancy and reportedly to have
hypertension before. Which of the following drugs is absolutely
contraindicated in pregnancy?

A. Prazosin
B. Captopril
C. Atenolol
D. Propranolol
E. Methyldopa

Question 18

Microscopic feature of endometrial curettage from a 26-year-old woman


who has primary infertility shows tortuosity of glands with epithelial lining
folds. Glandular secretion lessens. Stroma cells become round with increased
eosinophilic cytoplasm. Which of the following phase is related to patients
condition?

A. Late secretory phase


B. Menstrual phase
C. Mid-secretory phase
D. Early secretory phase
E. Proliferative phase

Question 19

What is phase that happened on day 15 to 28 of the menstrual cycle?


A. Proliferative phase-follicular phase
B. Secretory phase-proliferative phase
C. Follicular phase-luteal phase
D. Luteal phase-secretory phase
E. Ovulation -Ovulatory phase

Question 20

Mrs X, a 28-year-old primigravida has leaking of fluid per vaginam for the
past 3 h at 32 weeks of gestation. Per speculum examination confirms
leakage of clear amniotic fluid per vaginam. She is clinically stable with no
signs of infection. Ultrasound shows a singleton fetus in cephalic presentation,
appropriate for gestation with normal liquor and Dopplers. What is the correct
statement that supporting premature rupture of the membrane was occured
in this case?

A. Rupture of the amniotic membrane before 37 weeks gestation


B. Rupture of the amniotic membrane after 20 weeks gestation
C. Rupture of the amniotic membrane before latent phase of labor
D. Rupture of the amniotic membrane before active phase of labor
E. Rupture of the amniotic membrane after 37 weeks gestation

Question 21

A 38-year-old nulliparous woman presents requesting a bilateral


salpingo-oophorectomy. Her mother died of ovarian cancer at the age 64,
and her sister at the age 48. There is no family history of other cancers. You
advise her that her risk of developing ovarian cancer is what percentage

A. 1–2%
B. 30–40%
C. 0,2
D. 0,07
E. >50%

Question 22

An intensive phototherapy was initiated to a newborn baby with


hyperbilirubinemia. You explain to the parents about the condition of the
baby. The parents were asking about how phototherapy can help reduce the
bilirubin level. What is the correct explanation?

A. The visible light in the blue region of the spectrum transforms


unconjugated bilirubin into bilirubin photoproducts, mostly isomers of
bilirubin, that is soluble
B. All of the above are correct
C. The visible light in the blue region of the spectrum can increase the
glucuronyl transferase activity
D. The visible light in the blue region of the spectrum can help reduced
haemolytic process
E. The visible light in the blue region of the spectrum can help increased
enterohepatic circulation

Question 23

A patient come the outpatient clinic after completing curretage and


diagnosed with hydatidiform mole. she is asking you about her condition.
Which of the following pathologic features is most helpful in distinguishing
complete hydatidiform mole from normal placenta?

A. absence of blood vessels


B. sex chromatin positivity
C. trophoblastic proliferation
D. hydropic degeneration of villi
E. cellular atypia

Question 24

A 2-day-old baby boy was brought to the hospital complaining of yellow skin
all over his body. Blood laboratory examination revealed only an increase in
indirect bilirubin. What is the most appropriate etiology of jaundice related to
the case?

A. Pre hepatic
B. Pre and post hepatic
C. Post hepatic
D. Pre and intra hepatic
E. Intra hepatic

Question 25

A newborn girl, born at 35 weeks gestation, is brought to the transitional


nursery at 1 hour of life after the nurse concerned about her breathing. Her
mother is a 27-year-old primigravid with no significant past medical history.
The baby delivered vaginally and was occurred for 20 hours. The doctors
diagnose wa neonatal sepsis Which one is the best answer of the neonatal
innate immunity

A. The ILCs numbers are increase so the metabolic regulation will also
increase
B. The NK cells will increase secretion of IFN gamma
C. Increase adhesion ability
D. The NK cells will depress inhibitory receptors
E. increase cytoplasmic granules and degranulation

Question 26

Maternal Toxoplasma gondii-specific IgG and IgM antibody levels and


neonatal T. gondii-specific IgG, IgM, and IgA antibody levels confirm that the
baby has congenital toxoplasmosis. The infant is asymptomatic and has
normal ophthalmologic, auditory, and neurologic evaluations. Which of the
following is include as clinical issues that are likely to develop in this infant
several months to years later?

A. Learning disabilities
B. Visual impairment
C. All of the above
D. ;None of the above
E. Hearing loss

Question 27

A surgical pathology specimen from a 24-year-old woman seen at a


reproductive medicine clinic demonstrates a ciliated columnar epithelium.
Which one of the following is the organ that told in the case?

A. ;Fallopian tube ini gasi? Ciliated iniiii


B. Vagina ini gasi? bkn
C. Ovary
D. Cervix
E. Endometrium

Question 28
A 24-year-old woman diagnosed with G2P0A0–term parturient, the doctor
give her . Which of the following is the pharmacological property of this drug?

A. It is contraindicated for peptic ulcer


B. It has a long duration of action
C. It increases frequency and force of uterine contraction ini? iyaa
D. Its mechanism of action is stimulate prostaglandin synthesis
E. The absorption is reduced by food

Question 29

Suppose a new treatment for pain relief is to be tested among patients with
disease X. The measure of pain relief will be the percent change in pain level
as reported by the patient after taking the medication for 1 month. What do
type II error mean in this situation?

A. To conclude that the treatment doesnt have an effect when in fact it


has an effect
B. To conclude that the treatment has an effect when in fact it doesnt
have an effect
C. To conclude that the treatment has an effect using inappropriate
statistical method
D. To conclude that the treatment has an effect when in fact it has an
effect
E. To conclude that the treatment doesnt have an effect when in fact it
doesnt have an effect

Question 30

Describe the structure and function of the organs of the female internal
genitals The female reproductive system functions to produce gametes and
reproductive hormones, just like the male reproductive system; however, it
also has the additional task of supporting the developing fetus and delivering
it to the outside world. Unlike its male counterpart, the female reproductive
system is located primarily inside the pelvic cavity. What are the female
gonads called?

A. ovaries
B. ova
C. ovum
D. oviducts

Question 31

A 47-year-old female comes with pain following sexual activity with her
husband. Additionally, she complained of foul-smelling vaginal discharge. A
cytology test of the cervix revealed abnormal cells affecting one-third of the
cervical epitheliums thickness. What is the diagnosis of the patient?

A. CIN 2
B. CIN 4
C. CIN 3
D. CIN 1
E. Cervical cancer

Question 32

Mrs A, a 37-year-old primigravid consult to you about her baby condition, she
was felt the water is broke 2 hour ago. She did not felt any fever, the
movement of the baby is good. What is the best examination should be
performed to determine the diagnosis in the patient above?
A. Ferning test
B. Vaginal touche
C. Leopold test
D. Physiological examination
E. Per speculum

Question 33

A 47-year-old woman complains of postcoital bleeding, nearly as heavy as


menses. Which of the following is the most likely cause of the bleeding?

A. cervical ectropion
B. cervical carcinoma
C. cervical polyps
D. cervical nabothian cysts
E. cervical infection

Question 34

Mrs. Mei, a 28-year-old, G1P0A0, 37 weeks pregnancy was brought to the ER


with a chief complaint watery discharge from the vagina in the past one day
ago. She admitted that she had abdominal pain/cramp for several hours and
she felt the baby was still moving.Three days ago, she had experienced some
frequent, urging and painful voiding. She also had yellowish discharge from
the vagina.
Physical Examination reveals:
● Temperature is 37.8 oC,
● BP: 110/70 mmHg,
● HR: 92 bpm,
● RR: 24 x/m.
● Head and Neck are within normal limits
● Lung and heart are normal
● Suprapubic tenderness: (-)
● Costovertebral angle tenderness: (-)
● Obstetric examination:
● External examination :
● Fundal height: 34 cm above the symphysis
● Leopold maneuver :Head above symphysis,2/5, back on the right
● Fetal heart rate :156 bpm regular
Uterine contraction : twice every 10 minutes regular, 25 seconds, weak. Based
on the above patients condition, which of following is indicative of the third
trimester of pregnancy ?

A. Braxton Hicks contractions


B. Implantation
C. Initial fetal movement
D. Fetal head descent

Question 35

An 18-year-old patient delivered a non viable baby 3 days before. She


complained about pain and redness on her breast. What is the best method
to supress breastmilk after birth?

A. All of the above


B. Intra uterine device
C. combination of Contraceptive pills
D. Progestin only contraceptive pills
E. Implant

Question 36
A newborn girl, born at 35 weeks gestation, is brought to the transitional
nursery at 1 hour of life after the nurse concerned about her breathing. Her
mother is a 27-year-old primigravid with no significant past medical history.
The baby delivered vaginally and was occurred for 20 hours. The diagnose
was neonatal sepsis. What happened soon after birth in the neonates
immunology system?

A. The antibodies increase twice the original level


B. Decrease in gamma globulin
C. The neonate will form antibodies of its own to a significant extent.
D. The neonate loses a great degree of immunity from the mother
E. The babys own immune system begins to form antibodies and returns
essentially to normal by the age of 6 to 7 months.

Question 37

A -24-year-old woman is diagnosed G2P0A0 –term parturient. The doctor


gives her an oxytocin infusion. Which one of the following is the most
appropiate should be monitored?

A. Post partum bleeding


B. Fetal heart rate
C. Respiratory rate
D. Blood pressure
E. Nausea and vomiting

Question 38

A 45-year-old woman complains of pelvic pressure and abnormal uterine


bleeding. Ultrasound reveals an enlarged uterus with an intramural 4 cm
mass. Which of the following is the most common uterine neoplasm?
A. Adenomyosis
B. Choriocarcinoma
C. Leiomyoma
D. Adenocarcinoma
E. Sarcoma

Question 39

An 18-year-old patient finally delivered a 4,000-g infant vaginally. Her


prenatal course was complicated by anemia, poor weight gain, and maternal
obesity. Her labor was protracted, including a 3-hour second stage, a
mid-forceps delivery with a sulcus laceration, and a third-degree episiotomy.
She develops a persistent fever of 38,5 0C on the third day postpartum. What
is the most likely etiology? what is the most likeliy diagnosis of the patient?

A. Cholesistitis
B. Retention Fever
C. Peritonitis
D. Mastitis
E. Endometritis

Question 40

A 21-year-old pregnant woman was diagnosed as G2P1A0 18-20 weeks with


iron deficiency anemia. The doctor treated her with ferrous ascorbat. Which of
the following is the most often side effect of this preparation?

A. Premature contraction
B. Hematology disorder
C. Fatty liver
D. Peptic Ulcer
E. Kidney stone

Question 41

Following an uncomplicated vaginal delivery, when can women be advised to


resume coitus based on

A. 6 weeks postpartum
B. 3 weeks postpartum
C. 4 weeks postpartum
D. 2 weeks postpartum
E. 1 weeks postpartum

Question 42

A primiparous term pregnancy come with a blood-like fluid come with her
vagina. The symptomps felt for actually 2 days before. The patient didnt had
any sudden fluid flow from vagina. The fetal movement are still active. Her
vital sign are : BP 110/70 mmHg ; HR 88 bpm ; RR 20 bpm T 37,2oC. This is her
first experience. The midwife said that she is having a blood show. Following
statement is the most appropriate to explain the cause of the bleeding in this
patient

A. a result of placenta previa


B. not seen in breech presentation
C. vaginal bleeding as a result of trauma
D. a sign of impending obstetrical hemorrhage
E. a consequence of effacement and dilatation of the cervix

Question 43
A baby boy was born aterm, healthy, with body weight of 3000 grams. He and
his mom was discharged home at age 12 hours with the instruction to return
to pediatricians clinic in two days after discharge. After 2 days in home, he
was brought to clinic because the mother had noticed that her son is appear
more yellowish and weak, but still able to breastfeed.

Physical exam of the baby reveals:


● Vital signs: within normal limit
● Head: icteric sclera
● Abdominal wall: smooth, non-distended, liver and spleen: normal
● Extremities: jaundice
● Laboratory findings:
● Hb: 13 %
● Leucocyte: 15.000/mm3
● PCV: 42%
● Platelet count: 200.000/mm3
● Total bilirubin: 13 mg/dL
● Direct bilirubin: 0,4 mg/dL
● Babys blood group: O -
● Mothers blood group: A +
Coombs test: negative What is the etiology of neonatal hyperbilirubinemia
above?

A. ABO incompatibility
B. Rh incompatibility ?
C. Physiological jaundice
D. Neonatal hepatitis
E. G6PD enzyme deficiency

Question 44
A 2-day-old newborn female, brought by her mother to the clinic due to the
chief complaint of yellowish discoloration of her entire body. Initially, the
yellow color started appearing since birth. As the doctor in the clinic, you
suggest to do phototherapy. Which one that is the minor side effects of
phototherapy?

A. Retinal damage
B. None of the above
C. Skin rash
D. Delayed development
E. Bronze baby syndrome

Question 45

A 35-year-old woman came to the clinic for routine health screening. On


physical examination, the cervix was tested with acetic acid and the color
turned whitish. the doctor consider that she is having a cervical cancer in the
early stages. Which of the following is the most common symptoms
regarding the condition of disease that considered by the doctor?

A. Abdominal cramps and diarrhea


B. Vaginal bleeding???
C. Nausea
D. Anemia and extreme tiredness
E. Usually there are none symptoms??? → inii bener early stage

Question 46

A 25-year-old female came to the clinic complaining of genital itching for a


week. She complained her outer genital on the front to middle part were very
itchy Which nerve most likely carries the itching sensation?
A. Branch of ilioinguinal nerve
B. Branch of Inferior clunial nerve
C. Branch of Perineal nerve
D. Branch of genitofemoral nerve
E. Branch of Pudendal nerve

Question 47

A 33-year-old G2P1 is undergoing an elective repeat cesarean section at


term. The infant is delivered without any difficulties, but the placenta can not
be removed easily because a clear plane between the placenta and the
uterine wall cannot be identified. The placenta is removed in pieces. This is
followed by uterine atony and hemorrhage. What is the most appropiate
diagnosis of this case?

A. Placenta previa
B. Fenestrated placenta
C. Placenta accreta
D. Succenturiate placenta
E. Vasa previa

Question 48

Describe the structure and function of the organs of the female internal
genitals Structurally, the urogenital triangle is complex, with a number of
fascial layers and pouches.

A. contains part of the urethra, external urethral sphincter, and the vagina
B. a layer of tough fascia, between deep pouch and superficial pouch
C. a potential space between the superficial perineal fascia
D. Which of the following statement is the most appropriate regarding the
perineal membrane
E. a potential space between the deep of the pelvic floor

Question 49

which is most correct answer for ovarian ligament?

A. is a remnant of the superior part of the ovarian gubernaculum of the


fetus
B. is a double layer of peritoneum (mesentery) that extends from the sides
of the uterus to the lateral walls and floor of the pelvis
C. Connects the distal (uterine) end of the ovary to the lateral angle of the
uterus, just inferior to the entrance of the uterine tube
D. tethers the ovary to the anterior abdominal wall
E. is the largest part of the broad ligament, inferior to the mesosalpinx and
mesovarium

Question 50

A woman with G3P0A2 32nd weeks pregnancy come with complaint of


preterm labor. The amnion membrane are still intact and fetal movement is
good. Based on current clinical opinion, what is the management should be
included prior to 34 weeks of gestation?

A. Bed Rest
B. Antimicobial prophylaxis and Magnesium suphate to prevent infection
C. Amnioinfusion
D. Consideration of tocolytics and misoprostol to delay delivery
E. Corticosteroid administration
Question 51

A 25-year-old pregnant woman with 20 weeks of gestation goes to the


primary health center. The test results showed normal fetal growth but the
mothers body weight had not reached the target. Mothers are given advice
on nutritional intake to pursue weight gain and are given iron supplements
and folic acid. how much advice on folic acid supplementation for the
mother?

A. 1000 mcg
B. 100 mcg
C. 200 mcg
D. 400 mcg
E. 600 mcg

Question 52

A 20-year-old patient complained about foul smelled vaginal discharge


since 1 month ago. She works as a sex worker in one of red district in the city
and has a history of promiscuity. Vaginal discharge is tested and found gram
negative bacteria with intra celullar body. What is the main risk factor for this
patients illness?

A. Gender
B. Promiscuity
C. Genetic
D. Hygiene
E. Age

Question 53
A 24-year-old woman come to the clinic of obstetric and gynecology with her
husband, she has late menstruation period for about three weeks. They are
longing for children; the doctor made a pregnancy test. What are the sample
for measure a beta HCG level as an indicator of pregnancy?

A. Urine, serum
B. Urine, amniotic fluid
C. Urine, Amniotic fluid
D. Amniotic fluid, serum
E. whole blood, plasma

Question 54

A 26-year-old pregnant woman with 36 weeks of gestation goes to the clinic


for pregnancy control. The due date will be next 1 week. Pre pregnancy body
weight was 56 kg and mother’s height is 165 cm. What is acceptable body
weight target for the mother?

A. 63 kg
B. 61 kg
C. 59 kg
D. 75 kg
E. 70 kg

Question 55

What is the most accurate way of dating a pregnancy?

A. Last menstrual period


B. 1st-trimester ultrasound
C. 2nd-trimester ultrasound
D. 38 weeks
E. Last menstrual period and a 2nd-trimester ultrasound

Question 56

A term baby was born spontaneously from G2P1 mother. He was immediately
cried with good tones and did not need any rescucitation. After birth the baby
got early inisiation of breastfeeding. During pregnancy mother always in a
good condition and rountinly check up to the obstetrician. The doctor wants
to make management plan for the baby. When the best periode to screen
congenital hypotiroid?

A. More than 48 hours


B. Less than 24 hours
C. 24 hours to 72 hours
D. More than 72 hours
E. 24 hours to 48 hours

Question 57

An 18-year-old patient delivered a 3,500-g infant vaginally. There are no


abnormality of physical examination except anemic in conjunctiva.
Laboratory findings :
● Haemoglobine level : 10 gram/dl
● Leucocyte : 7.000/ mm3
● PCV : 22 %
● Platelet count : 180.000 / mm3
● MCV : 70 fL
● MCH : 24 pg
● MCHC : 30 g/dL
The following statement is correct about the blood values changes in
postpartum phase

A. Immediately following delivery, there is slight decrease of blood volume.


Blood volume returns to nonpregnant level by the 2nd week.
B. Cardiac output decreases soon after delivery to about 80% under the
prelabor value but slowly returns to normal within 4 week.
C. Platelet count increases soon after the separation of the placenta.
D. RBC volume and hematocrit values returns to normal by 2 weeks
postpartum after the hydremia disappears.
E. Leukocytosis to the extent of 60,000/mm3 occurs following delivery
probably in response to stress of labor.

Question 58

A 37-year-old woman is pregnant with her third child. Her previous children
were both born by vaginal delivery. First trimester ultrasound confirmed her
menstrual dates and she is now 37 weeks. At her last appointment at 36
weeks gestation the doctor suspected that the baby was in breech
presentation.

Her blood pressure 130/80 mmHg and abdominal examination suggests a


breech presentation with the sacrum engaged. Estimated fetal weight 3 kg.
AFI 18 cm.

If you want to do vaginal breech delivery, which one followed is the right
steps?

A. engagement – flexion – descent – internal rotation – extension –


external rotation – expulsion
B. engagement – descent – flexion – external rotation – extension –
internal rotation – expulsion
C. engagement – descent – internal rotation - flexion – extension –
external rotation – expulsion
D. engagement – descent – flexion – internal rotation – extension –
external rotation – expulsion
E. engagement – flexion – descent – internal rotation – external rotation –
extention - expulsion

Question 59

A 19-year-old woman presented to the clinic with the primary complaint of


vaginal discharge that had been present since the previous week. She
confessed to being a promiscuous woman. The doctor found a frothy cream
discharge under the speculum and a motile protozoan with whip-like tail and
with jerky, twisting locomotion activity . What is the infected stadium of the
pathogen in the case?

A. Sporozoit
B. Cyst
C. Trophozoit
D. Egg
E. Larvae

Question 60

Suppose serum-cholesterol levels in spouse pairs (husband and wife) are


measures to determine whether there is correlation between cholesterol
levels in spouses. Suppose that the coefficient of correlation is 0.31 based on
51 spouse pairs. What is the value of the test statistics?

A. 2.26
B. 2.33
C. 2.35
D. 2.28
E. 2.31

Question 61

At 39 weeks gestation, a fetus was felt to be in breech presentation as judged


by information gained through Leopolds maneuvers. The breech was well
down in the pelvis, and the uterus was irritable. Pelvimetry was within normal
limits and the estimated fetal weight was 4000 kg What is the best
management should be conducted for this patient?

A. expectant management
B. cesarean section
C. external cephalic version
D. internal podalic version
E. oxytocin induction

Question 62

Describe the structure and function of the organs of the female internal
genitals The female reproductive system functions to produce gametes and
reproductive hormones, just like the male reproductive system; however, it
also has the additional task of supporting the developing fetus and delivering
it to the outside world. Unlike its male counterpart, the female reproductive
system is located primarily inside the pelvic cavity.

What is the name of structure who serve as the conduit of the oocyte from the
ovary to the uterus?

A. fallopian tubes
B. fornix
C. vagina
D. ampulla

Question 63

Question textA 25-year-old woman has had a grayish vaginal discharge with

a fishy odor for three days. The discharge is inspected and found to be

positive for the whiff test. What is the most likely transmission method of the

disease ?

A. air borne
B. Sexual intercourse
C. food borne
D. water borne
E. Physical touch

Question 64

Cervical cancer was the leading cause of death in North American women.
Cervical cancer also has high rate of metastase. which is the nearest cervical
cancer spread through the blood?

A. via the ovarian arteries into the abdominal aorta


B. via the uterine venous plexus drain into the internal iliac veins
C. via the ovarian vein drain into the inferior vena cava
D. via the pampiniform plexus of vein drain into the left renal vein
E. via the uterine arteries into the right superior mesenteric vein

Question 65
A 24-year-old patient came to primary health facility to ask about
emergency contraceptives. She had unprotected sexual intercourse two days
before. She is given pills that contains 0,75 mg of levonogestrel. Which is the
best method to consume levonogestrell emergency contraceptives?

A. First pill is consumed immediately before 72 hours unprotected sexual


intercourse, second pill is consumed 24 hours after the first pill
B. First pill is consumed immediately before 72 hours unprotected sexual
intercourse, second pill is consumed 72 hours after the first pill
C. First pill is consumed immediately before 72 hours unprotected sexual
intercourse, second pill is consumed 36 hours after the first pill
D. First pill is consumed immediately before 72 hours unprotected sexual
intercourse, second pill is consumed 6 hours after the first pill
E. First pill is consumed immediately before 72 hours unprotected sexual
intercourse, second pill is consumed 12 hours after the first pill

Question 66

A 30-year-old woman presents with menorrhagia. Blood assay reveals


increased level of 17-beta-estradiol. From which of the following
17-beta-estradiol is directly synthesized?

A. Androgen
B. Estrone bukan ini? stuju
C. Cholesterol
D. Androstenedione
E. Pregnenolone

Question 67
A 30-year-old woman presents for her annual examination. She inquired
whether she should received the HPV vaccine. Which population would
benefit from receiving this vaccine?

A. only virginal woman and who have never had an abnormal papsmear
B. men
C. woman all ages
D. 9-26 years
E. pregnant woman or lactatinf woman

Question 68

Bob and Miriam Pataki walk in to your office with their 16 year-old daughter

Nina, who has not yet menstruated for the first time. Examination revealed a

thin girl with normal breast development and the normal presence of pubic

and axillary hair. On talking with Nina, you discover she is a junior in high

school and is very active in sports. She is the highschools track and field star,

and she is an avid swimmer. Because of her track practice, Nina stays up late

most nights to do her homework, consequently, she only gets five to six hours

of sleep a night. What is the most likely diagnosis for Nina?

A. exercise induced amenorrhea


B. excessive secretion of melatonin
C. adrenal hyperplasia
D. testicular feminization

Question 69

A primiparous with eclampsia is being threated in emergency ward. The


consultant on duty explain to the student that this condition is happened due
to generalized vasoconstriction. These following complications is related to
pathophysiology of the generalized vasoconstriction in preeclampsia :

A. placental abruption
B. intrauterine growth restriction fetus
C. acute uteroplacental insufficiency
D. chronic uteroplacental insufficiency
E. All of the options are true

Question 70

A woman 29 years old had a positive pregnancy test 3 days ago after she
realized that she had missed a period. She had had regular cycles for 5 days
every 28 days. Her body weight is 40 kg with height 150 cm. This is her first
pregnancy. She has a platypelloid pelvic type. How should she delivered?

A. Further examination needed ini bukan?


B. She need to do caesarian section di google hrs cesar krn lobangnya
sempit?/ sbnrnya bisa vaginal tp susah gt kt gugel
C. She can try vaginal birth first then do caesarean if necessary
D. She can choose any type of birth she wants
E. She can try to give vaginal birth

Question 71

A 26-year-old pregnant woman goes to the clinic for 4th week pregnancy
control. Pre pregnancy body weight was 56 kg and mother’s height is 165 cm.
How much recommended body weight in 12th weeks of pregnancy?

A. 56 kg
B. 65 kg
C. 63 kg
D. 60 kg
E. 58 kg

Question 72

A 37-year-old G2P1A0 was found to acquire active toxoplasmosis in her 16


weeks of pregnancy. Her laboratory results showed : IgM Toxoplasma (+), IgG
Toxoplasma (+), and PCR Toxoplasma (+).No abnormality of the fetus was
found on USG that performed before and after spiramycin therapy take by the
mother. She gave spontaneous birth and recovers well, but the baby was
born with sepsis and has a congenital infection of toxoplasmosis. What is the
pathogens organelle that play role in the attachment and invasion of the host
cells in the case above?

A. Microneme and Rhoptry


B. Dense granule and nucleus
C. Outer membrane and citoplasma
D. Apicoplast and chromatin
E. Polar ring and cytoplasm

Question 73

A 37-year-old G2P1A0 was found to acquire active toxoplasmosis in her 16


weeks of pregnancy. Her laboratory results showed : IgM Toxoplasma (+), IgG
Toxoplasma (+), and PCR Toxoplasma (+).No abnormality of the fetus was
found on USG that performed before and after spiramycin therapy take by the
mother. She gave spontaneous birth and recovers well, but the baby was
born with sepsis and has a congenital infection of toxoplasmosis. What is the
most likely animal involved in the above case?
A. Dog
B. Cat
C. Cow
D. Mosquito
E. Bat

Question 74

Bob and Miriam Pataki walk in to your office with their 16 year-old daughter
Nina, who has not yet menstruated for the first time. Examination revealed a
thin girl with normal breast development and the normal presence of pubic
and axillary hair. On talking with Nina, you discover she is a junior in high
school and is very active in sports. She is the highschools track and field star,
and she is an avid swimmer. Because of her track practice, Nina stays up late
most nights to do her homework, consequently, she only gets five to six hours
of sleep a night. What is condition that describe Ninas problem ?

A. hypogonadotropic hypogonadism
B. hypogonadotropic hypergonadism
C. hypergonadotropic hypergonadism
D. hypergonadotropic hypogonadism

Question 75

The parents ask about future pregnancies and whether they have an
increased risk of having another child with Down syndrome. You recall that
the mother and father are 27 and 29 years old, respectively. Select the true
statement below about recurrence risk and trisomy 21 syndrome:

A. Fifty percent of new diagnoses are sporadic.


B. If both parents have normal karyotypes, recurrence risk is 10%.
C. Ninety-five percent of cases are sporadic
D. None of the above
E. Recurrence risk is higher in a 45-year-old woman than for a
25-year-old woman with a balanced translocation.

Question 76

A 30-year-old in 6 months pregnancy come for prenatal care and


ultrasonography examination. Examination result come with a difference
between ultrasound weeks and menstrual period weeks of pregnancy. the
ultrasound result are bigger. Which of the following biometrics parameter
most accurately predicts gestational age in the second trimester ?

A. Abdominal Circumference
B. Head Circumference
C. Biparietal diameter
D. Humerus Length
E. Femur length

Question 77

A 56-year-old patient presents with enlarged abdomen and difficulty in


breathing since 5 months ago. She lost 5 kg of her weight, and she always feel
tired, bloated, and nausea. She has difficulty in defecating and urinating What
is the majority of symptoms on early stage of the disease?

A. Non specific
B. Difficulty in urinating
C. Enlarged abdomen
D. Difficulty in breathing
E. Heavy menstrual bleedomh
Question 78

A 24-year-old G1P0A0 patient came to primary health facility to do annual


antenatal care check up. She is prescribed folic acid tablets and ferrous
tablets by the doctor Which of the following nutrients is most likely to be
deficient during pregnancy?

A. Vitamin D
B. Vitamin A
C. Iron
D. Calcium
E. Folic acid

Question 79

A baby with AB blood group was born at term gestation. Her mother has
blood group O, and Rh negative. On admission to the nursery, the nurses
thought that the babys skin had a yellow tinge. The physician believed this
was only very mild jaundice and chose to ignore it. An astute nurse, however,
took an STB at age 12 hours, the result of which was 9.2 mg/dL. Not very high,
responded by the physician. By the next day (28 hours) the STB value was 15
mg/dL. What would you do?

A. Baby can go home and continue breast feeding


B. Place the infant under intense phototherapy and repeat the STB in 4 to
6 hours
C. Observe the baby and repeat the STB in another 24 hours
D. Directly exchange transfusion
E. Begin phototherapy and proceed to exchange transfusion

Question 80
A 10-day-old newborn need red cell transfusion for massive bleeding due to
Vit.K deficiency. The result of blood grouping test showed: Anti-A: Negative,
Anti-B: Negative, Anti-AB: Negative, Anti-Rh: Positive What is the blood group
of the newborn?

A. A Rh (-)
B. O Rh (-)
C. A Rh (+)
D. ;AB Rh (+) ini kan maksudnya?
E. O Rh (+)

Question 81

A 22-year-old P2A0 patient came to primary healthcare facility with breast


pain. She delivered a 2500-g infant vaginally three days ago. She has 38oC
fever, tenderness and redness on her right breast What is the correct
diagnosis?

A. Mastitis
B. Breast cancer
C. Galactocele
D. Breast engorgement
E. Breast Abcess

Question 82

An 18-year-old patient finally delivered a 4,000-g infant vaginally. Her


prenatal course was complicated by anemia, poor weight gain, and maternal
obesity. She also complained that the production of her breastfeeding milk is
not much. On abdominal palpation, the uterine fundus is soft and at the level
of umbilicus. Which of the following statement is true about lactation?

A. Human chorionic somatomammotropin that is secreted by placenta


has lactogenic properties.
B. Immediately after the baby is born, the sudden increased of estrogen
secretion allows the lactogenic effect of prolactin
C. Soon after parturition, the pituitary begins to secrete sufficient
gonadotropic hormone
D. Oxytocin has a role in milk ejection and is one of the anterior pituitary
hormones
E. Prolactin is a hormone that promotes lactation and its production is
inhibited by hypothalamus

Question 83

What are the important component of Kangaroo Mother Care


A. Exclusive breast feeding and cord care
B. kangaroo mother care is not important
C. Cord care and hygine
D. Skin to skin contact and exclussive breast feeding
E. Skin to skin contact and hygine

Question 84

You are examined a term patient in the labor and delivery suite What regimen
is recommended for PROM,caused by Group B streptococcus?

A. Metronidazole
B. Cephalosporin 3rd generation
C. Pirymethamin
D. Cephalosporin 1st generation
E. Doxyciclin

Question 85

The investigators want to compare cholesterol level of the children whose


fathers have died from heart disease versus the children whose fathers do
not have a history of heart disease. Suppose from 100 children whose fathers
have died from heart disease, the observed mean and standard deviation of
the cholesterol level are 207.3 mg/dl and 35.6 mg/dl. Another 80 children
whose fathers do not have a history of heart disease are also identified and
the observed mean and standard deviation of the cholesterol level are 193.4
mg/dl and 17.3 mg/dl. It is assumed that the variances of the two groups are
not equal. One medical student wants to test a hypothesis that birthweight of
an infant is associated with smoking status of the mother during first
semester of pregnancy. She records birthweights of infants (grams) and
smoking status of the mother and divides the smoking status into four
groups: non-smoker, ex-smoker, current smoker and smokes less than 1 pack
per day, and current smoker and smokes 1 pack per day or more. If the
assumptions for parametric test are not met, which of the following is the
correct alternative test?

A. .Wilcoxon sum-rank test


B. Kolgomorov – Smirnov test
C. Wilcoxon signed-rank test
D. Kruskal – Wallis test
E. Friedman test

Question 86

A baby 2500 gr baby boy was born from 29 y.o, primigravida mother,
following toxoplasmosis infection that occured 19-20 weeks pregnancy.
During physical exam the baby found that macrochepaly and congenital
cataract. What is the mode of transmission of the infection occurred in this
case?

A. Delivery an infected tissue


B. Hematogenesis spread like bacteria
C. Transplacental
D. Sexual intercourse during pregnancy
E. Ascending passage of a virus

Question 87

A 35-year-old woman, P2A0 comes to the gynaecological clinic with the chief
complaint fever since 4 days before, after delivering a baby girl 6 days prior,
with a normal spontaneous vaginal delivery at term She also felt slightly
bilateral lower abdominal tenderness .On laboratory exam, she was found to
have a normal amount of foul-smelling lochia. The labs showed an elevated
white blood cell (WBC) count and anemia. 1. Following organ associated with
the chief complaint , EXCEPT :

A. Involution uterine
B. Remodelling cervix
C. Urine retention
D. Constriction salphyng (cava)
E. Edematous vagina and vulva

Question 88

A 64-year-old woman presented with abdominal bloating, low back pain,


early satiety and progressive fatigue. She underwent hysterectomy 5 years
ago for benign indication; hypothyroidism managed medically; generalized
anxiety disorder managed medically. She has 5 children. Her step mother
died with breast cancer. On physical examination, there was found diffuse
lumbosacral pain with movement; abdominal tenderness and significant
abdominal distension with a fluid weight consistent with ascites; unintentional
weight loss of 4 kgs. She was diagnosed with ovarian cancer. Which of the
following condition of this patient that has most associated for developing
her disease?

A. Her age
B. her underweight
C. the family history of breast cancer
D. Having multiple children
E. her thyroid problems

Question 89
A 32-year-old woman presents at her physicians office complaining of
nausea and vomiting. The history reveals that her symptoms have been
present for over a month and that they seem to be worse in the morning. A
urine sample is taken and shows that the woman is pregnant Which of the
following physiological changes that occur in this woman now?

A. Increased conversion of glucose to glycogen


B. Reduced circulating gonadotropin levels
C. hyperventilation leading to increase tidal volume
D. Decreased production of cortisol and corticosterone

Question 90

A 13-year-old boy was involved in a traffic accident and was taken to the
hospital. The patient requires a blood transfusion. Blood group revealed B Rh+
blood group. What is the blood group that acts as universal donor for the
patient?

A. O
B. AB
C. B
D. A
E. D

Question 91

A 27-year-old female came to Dermato-venereology clinic because of


vaginal discharge. The symptoms had first appeared since 2 weeks. The
patient had history having sexual intercourse without using condom. The
complaint accompanied by dyspareunia and dysuria. Venereological
examination revealed strawberry cervix with yellow-green frothy vaginal
discharge. Which of the following is a first line treatment for patient?

A. Cefixime
B. Ceftriaxone
C. Metronidazole
D. Doxycycline
E. Erythromicine

Question 92

In simple linear regression, to test the hypothesis H0: β1 = 0 versus H1: β1 ≠ 0,


with significance level α, which of the following is the correct decision rule to
reject H0 using F test?

A. F > F1, n – 4, 1 – α
B. F > F1, n – 2, 1 – α
C. F > F1, n – 5, 1 – α
D. F > F1, n – 3, 1 – α
E. F > F1, n – 1, 1 – α

Question 93

A 45-year-old woman went to primary health care presents with pain after
sexual intercourse with her husband. She also complained about foul smelled
vaginal discharge. She was examined and diagnosed as cervical cancer
Which of the following stage can be treated with surgery?

A. Stage IIIIB
B. Stage I
C. Stage IIIA
D. Stage IIIIC
E. Stage IV

Question 94

Which of the following is the correct property of correlation coefficient?

A. Variable X and Y can be interchangeable without affecting its


magnitude
B. Its magnitude indicates how close the points are to the curve line
C. Correlation between x and y does necessarily imply a cause – effect
relationship
D. Its sign indicates the proportion of the variability of the relationship
E. Its value is valid only within the range of values of X and Y in the
population

Question 95

Non-initiation of resuscitation in the extremely immature infant or in cases of


severe congenital anomaly is a challenging problem in neonatology.
According to guidelines from the American Heart Association and American
Academy of Pediatrics, non-initiation of resuscitation appears appropriate
in....

A. Gestation <23 weeks or birthweight <500 g


B. Gestation <23 weeks or birthweight <500 g
C. Gestation <23 weeks or birthweight <400 g
D. Gestation <25 weeks or birthweight <400 g
E. Gestation <25 weeks or birthweight <500 g

Question 96
A newborn girl, born at 35 weeks gestation, is brought to the transitional
nursery at 1 hour of life after the nurse is concerned about her breathing. Her
mother is a 27-year-old primigravid and she had fever four weeks before
delivery. The baby delivered vaginally and was occurred for 20 hours. The
birth weight is 2200 grams. The working diagnosis for the baby is neonatal
sepsis. Which is true about immune system in a newborn baby?

A. the immunity of the newborn baby age 1 week is low


B. soon after the baby is born, the babys antibody is formed
C. the gamma globulin concentration will reach the normal level by the
age of 1 month
D. By the end of the first month, the babys gamma globulin will increase.
E. eczema is often happen in newborn infants.

Question 97

A woman of 70 years old complained of vaginal bleeeding with a


pedunculated, fragile mass from the cervix. The pathological finding is
polygonal, atypical tumor cells that invade the fibrous stroma of keratin pearl.
Which of the most appropriate diagnosis of this patient?

A. Cervical adenocarcinoma
B. CIN III
C. Cervical polyp
D. Cervical squamous cell carcinoma
E. Microinvasive Ca Cervix

Question 98

In simple linear regression, which of the following is true if H0: β1 = 0 is not


rejected? Yang manaa
A. Assuming that we have not made a type II error, There is a good fit of
the data to a linear model, but some curvilinear model might provide
an even better fit
B. Assuming that we have not made a type II error, the relationship
between X and Y is not linear. Some curvilinear model provides a better
fit to the data
C. Assuming that we have made a type II error, the relationship between X
and Y is not linear. Some curvilinear model provides a better fit to the
data
D. Assuming that we have not made a type II error, The relationship is
linear and sufficient strength to justify the use of sample regression
equation to predict and estimate Y for given value of X
E. ;Assuming that we have made a type II error, although the
relationship between X and Y may be linear it is not strong enough for
X to be of much value in predicting and estimating Y

Question 99

A 27-year-old woman has left ovarian tumor. She was done the ovarectomy
and the tumor sent to Pathology lab. Macroscopically shows cystic mass,
diameter 7 cm and its content of chocolate color mass. Microscopically
shows ovarian stromal lined by ulcerated epithelial surrounding by
hemosiderin laden macrophage. Which is the most likely diagnosis ?

A. Follicular cyst
B. Endometriosis cyst kayanya ini
C. Paraovarial cyst
D. Simple cyst
E. Luteal cysrt

Question 100
Mrs. Yanti 30 years, had been married for 3 years and has not had any
children yet. She visit obgyn clinic. From general physical examination there
are no abnormalities found. From USG examination, there is a blockage in her
uterine tube due to infection, and disturbance in motility. What is the
histological feature that is responsible in uterine tube motility : What is the
histological feature that is responsible in uterine tube motility :

A. Stereocilia
B. Cilia
C. Microcillia
D. Villi
E. Microvilli

Question 101

A female infant born at 36 weeks of gestation following prolonged rupture of


the membranes (5 days). The mother had fever during delivery and several
days before. Spontaneus delivery was helped by a midwive. The nurse in the
recovery room just observed the baby had an apneic episode with skin color
change that required stimulation. The baby is pink and responsive to
examination but has a respiratory rate of 70 breaths per minute. What is the
most possible etiology of the case?

A. Escherichia coli
B. Haemophilus spp.
C. Listeria monocytogenes
D. Klebsiella spp
E. Enterobacter spp

Question 102
What has developed at the ends of respiratory bronchioles and lung tissue is
well vascularized?

A. Primordial alveoli
B. Primordial alveolar ducts
C. Lumina of bronchi
D. Terminal bronchioles
E. Respiratory bronchioles

Question 103

At birth, the lungs are approximately half filled with fluid. What route the fluid
in the lungs is cleared at birth?

A. By Swallowed
B. Into the cardiac circulation
C. By pressure on the thorax
D. By Inhalation
E. Into the pulmonary circulation

Question 104

Describe the structure and function of the organs of the female internal
genitals In clinical practice, the term perineum is frequently used to describe
the area between the external genitalia and the anus. However, in anatomical
terms, the perineum is a diamond-shape structure. Which of the following
structure divides the perineum into urogenital and anal triangles?

A. ischial tuberosities
B. ischial fossa
C. ischial transverse
D. ischial tubercle

Question 105

A 30-year-old woman attend a routine antenatal appointment at 31 weeks


gestation. This is her third pregnancy. All her previous children delivered
spontaneously via vaginal deliveries at term. Her second child 18 months old
and the delivery was complicated by post partum haemorrhage requiring 3
unit blood transfusion. This pregnancy has been uncomplicated so far with
normal blood tests, normal 11-14 week ultrasound and normal anomalu
ultrasound scan. She feels generally tired. She reports good fetal movements
(more than 10 per day).

Blood pressure 125/70. Her laboratory test result taken at 28 weeks is Hb


7.8g/dL, MCV 68 fl, Leucocyte 11.2x109/L, platelets 237x109/L, urinalysis
negative. Blood type A negative.

How will you manage this woman for the last trimester of pregnancy?

A. Prescribed oral ferrous sulphate 200 mg twice daily


B. Prescribed oral ferrous sulphate 200 mg twice daily + folic acid 400
mcg once daily
C. Considered blood transfusional injections immediately
D. Prescribed parenter
E. Do further investigations but still prescribed oral ferrous sulphate

Question 106

A full-term male infant is born via spontaneous vaginal delivery to a


27-year-old primigravida woman. Her pregnancy was unremarkable, and she
declined prenatal testing. As the doctor covering the delivery room, you are
called to evaluate the infant shortly after birth because of a concern for
dysmorphic features. On your examination, you find that the baby has
features consistent with trisomy 21 syndrome. Which feature is consistent with
your finding?

A. Transverse palmar crease, cutis aplasia, polydactyly, cleft lip/ palat;


micrognathia, small mouth, malformed low-set ears, occipital
prominence
B. Hypoplastic nails, clenched hand, overlapping of second finger over
third finger or fifth finger over fourth finger, rocker- bottom feet,
micrognathia, small mouth, malformed low-set ears, occipital
prominence
C. Transverse palmar crease; flat facies; upslanting palpebral fis- sures;
rocker-bottom feet, micrognathia, small mouth, malformed low-set
ears, occipital prominence
D. Transverse palmar crease, cutis aplasia, polydactyly, cleft lip/ palate,
colobomas, narrow hyperconvex fingernails
E. Transverse palmar crease; flat facies; upslanting palpebral fis- sures;
wide gap between first and second toes; inner epicanthal folds; large
protruding tongue; nuchal redundancy; hypotonia

Question 107

A 25-year-old pregnant woman with 20 weeks of gestation goes to the


primary health center. The test results showed normal fetal growth but the
mother's body weight had not reached the target. Mothers are given advice
on nutritional intake to pursue weight gain and are given iron supplements
and folic acid. How is the nutritional advice for the mother?

A. Fulfill a balanced nutrition by increasing protein by 50 grams


B. Increase fruit and vegetable intake by 100 grams
C. Fulfill a balanced nutrition by increasing fat intake by 5 grams
D. Fulfill a balanced nutrition by increasing calorie by 300 kcal
E. Increase carbohydrate intake by 300 grams

Question 108

A 30-year-old woman at 16 weeks gestation presents for regular check up


with a BP of 144/95, no edema, no proteinuria, FHT 140. According to the recent
clinical studies, which of the following agent can prevent the development of
the conditions in this patient

A. Vitamin D
B. Vitamin C
C. Folic Acid
D. Acetosal
E. Magnesium

Question 109

A newborn girl, born at 35 weeks gestation, is brought to the transitional


nursery at 1 hour of life after the nurse concerned about her breathing. Her
mother is a 27-year-old primigravid with no significant past medical history.
The baby delivered vaginally and was occurred for 20 hours If we diagnose
this patient with neonatal sepsis what kind of management should be given?

A. Give IVIG
B. Antibiotic is given after blood culture result is revealed
C. Give surfactant
D. Postponed nutritional therapy
E. Empirical antibiotic therapy was given immediately
Question 110

The consistency of the cervical mucous is an indicator of phase of the


menstrual cycle and hence fertility. What is the hormon that impacts the
morphology of cervical mucous in the case?

A. LH
B. hCG
C. Progesterone
D. Estrogen and progesterone
E. Estrogen

Question 111

In a normal pregnancy the primary change in womans cardiovascular


system is a marked increase cardiac output What is the major cause of this
following conditions in the first half of pregnancy?

A. Increased systemic vascular resistance


B. increased heart rate
C. Increased duration of contraction
D. Decreased vascular resistance

Question 112

A baby with AB blood group was born at term gestation. Her mother has
blood group O, and Rh negative. On admission to the nursery, the nurses
thought that the babys skin had a yellow tinge. The physician believed this
was only very mild jaundice and chose to ignore it. An astute nurse, however,
took an STB at age 12 hours, the result of which was 9.2 mg/dL. Not very high,
responded by the physician. By the next day (28 hours) the STB value was 15
mg/dL. Which one of the following answer is correct about neonatal
hematopoiesis?

A. The red cells of the newborn are microcytic, MCV reaching adult values
by the 4th week
B. The blood volume per kilogram decreases over weeks.
C. hemoglobin and hematocrit values deflate in the first several hours
after birth
D. Red Blood charactertic : microcytic normochromic cells,
normochromasia, and a lot of nucleated red blood cells
E. By 3 -5 days after birth, nucleated red blood cells are normally found
in the blood

Question 113

A 30-year-old woman is 10 weeks gestation. She always attend pregnancy


dating regularly. She consume folic acid daily. Which disease can be
prevented with folic acid consumption?

A. neural tube defect


B. thalllasemia
C. cardiac abnormalities
D. down’s syndrome
E. phenylketonuria

Question 114

A 45-year-old woman went to primary health care presents with pain after
sexual intercourse with her husband. She also complains about foul smelled
vaginal discharge. Doctor advised her to do visual inspection with acetic as a
cervical cancer screening. Which of the most correct way to prepere the
solution?

A. Adding 10 ml glacial acetic acid to 95 ml distilled water


B. Adding 20 ml glacial acetic acid to 95 ml distilled water
C. Adding 2 ml glacial acetic acid to 98 ml distilled water
D. Adding 5 ml glacial acetic acid to 95 ml distilled water
E. Adding 1 ml glacial acetic acid to 99 ml distilled water

Question 115

A 28-year-old woman with a history of ectopic pregnancy presents for


contraceptive counselling. Which of the following contraceptive methods
would be contraindicated?

A. Oral contraceptives
B. ;minipills
C. Cervical cap
D. Male condoms
E. IUD

Question 116

Mrs A, a 30-year-old primigravida come to your hospital to consult about her


pregnancy. There no leakage of the amniotic fluid, the movement of the baby
is in good condition. Her last menstrual period is September 25, 2019. The date
of examination is March 9, 2020. The menstrual cycle is 28 days How old is the
gestational age at the time of examination?

A. 30-31 weeks of pregnancy


B. 26-27 weeks of pregnancy
C. 24-25 weeks of pregnancy
D. 22-23 weeks of pregnancy.
E. 28-29 weeks of pregnancy.

Question 117

On the fourteenth day of the menstrual cycle, the pituitary gland releases a
hormone which allows the ovary to release a matured egg. This egg is then
swept into the fallopian tube. What phase of the menstrual cycle
encompasses these events?

A. Luteal phase
B. Ovulatory phase
C. Menstrual phase
D. Follicular phase

Question 118

Pelvic Diaphragma and Neovasculature of Pelvis You are medical doctor


which on duty in obstetric and gynaecologic outpatient clinic. You will
determines the position of foetus and estimates the size, comparing the
anticipated birth weight with the pelvic structure of the mother. Which of the
following statment is the most unlikey correct regarding the features of the
female pelvis?

A. A narrow subpubic angle between inferior pubic rami compared to a


wide angle in the male
B. A wider inferior pubic angle than male
C. A shallow false pelvis with flaring of the ilia compared to the deep false
pelvis with more vertical ilia in male
D. a pelvic outlet larger than male

Question 119

A 42-year-old woman with a history of multiple sexual partners during her


lifetime is diagnosed with high-grade cervical intraepithelial neoplasia which
is strongly associated with oncogenic virus infection. Which of the following
statement is best explain the pathogenesis of the disease?

A. The virus is commonly causing an acute infection and develop a


high-grade epithelial lesion
B. The viral E4 protein is essential to induce cellular transformation
C. The virus is exclusively infecting epithelial cells of genital mucosa
D. The viral E6 protein is disrupting the p53 protein function
E. The viral E1 protein causing degradation of tumor suppressor genes

Question 120

A 27-year-old women came to the clinic for second visit continuing the
control of her anemia therapy. She had received iron supplementation for 3
months according to the dosage. Physical examination showed normal BMI
and anemia. Her blood laboratory examination showed low iron serum level
which is the same like her first previous result. What is the most correct
molecule involved and found in low level in her blood?

A. Erythropoietin
B. Vitamin C
C. ;Hepcidin
D. GDF-15
E. Albumin

Question 121
A 27-year-old female came to Dermato-venereology clinic because of
vaginal discharge. The symptoms had first appeared since 2 weeks. The
patient had history having sexual intercourse without using condom. The
complaint accompanied by dyspareunia and dysuria. Venereological
examination revealed strawberry cervix with yellow-green frothy vaginal
discharge. Which of the following is a first line treatment for patient?

A. Doxycycline
B. Cefixime
C. Ceftriaxone
D. Metronidazole
E. Erythromicine

Question 122

A 25-year-old woman use progestin injection as her choice of contraception.


She used to visit a clinic for the injection. She was supposed to be injected 3
days ago. What is she supposed to do regarding her missed injection?

A. She should take the injection as soon as possible


B. She must take progestin pills
C. The doctor must increase the injection dose
D. She must use emergency contraceptive
E. She must avoid sex

Question 123

A 20-year-old patient presents with cyclic, midline pain, low back pain and
pelvic fullness. Difficulty in defecating and urinating is also present. This
patient has not been able to have a mentrual period. A palpable abdominal
and pelvic mass is found during physical examination Which of the following
is the most likely diagnosis?

A. Meigg Syndrome
B. .Hematometra
C. Ovarian cyst
D. Leimyoma
E. Ovarian cancer kt tim mde

Question 124

A 35-year-old woman came to the clinic for routine health screening. On


examination, the citology result was found low-grade squamous
intraepithelial lesion. According to Bethesda Classification, what is the most
correct diagnosis of the patient?

A. LSIL
B. HSIL
C. AGC
D. SCC
E. ASC-US
Question 125

A 28-year-old woman came to out-patient department with pain in her tooth.


From oral examination she suffered caries dentist. Before taking further
treatment, the doctor gave her NSAID to reduce the inflammation and pain. It
was also known that the patient are 6-month old pregnant. Which one of the
following is the safest NSAID for this patient?

A. Mefenamic acid
B. Salicylate
C. Paracetamol
D. Ibuprofen
E. Metampyrone

Question 126

Define the pelvic girdle and describe the bones and ligaments of the pelvis

Explain the three regions of the hip bone and identify their bony landmarks

Describe the openings of the pelvis and the boundaries of the greater and
lesser pelvis The pelvic girdle (hip girdle) is formed by a single bone, the hip
bone or coxal bone or coxae (coxal = hip), which serves as the attachment
point for each lower limb. Each hip bone, in turn, is firmly joined to the axial
skeleton. The right and left hip bones also converge anteriorly to attach to
each other.

The hip bone, or coxal bone, forms the pelvic girdle portion of the pelvis. The
paired hip bones are the large, curved bones that form the lateral and
anterior aspects of the pelvis.
A. The slightly curved posterior margin of the ischium above the ischial
tuberosity is
B. lesser sciatic foramen
C. lesser sciatic notch
D. greater sciatic notch
E. sciatic canal

Question 127

A 28-year-old woman want to use progestin only pills to protect her from
pregnancy. Which of the following condition would be an absolute
contraindication for her to use it?

A. Having a diabetes
B. Breastfeeding
C. BMI 20-35
D. Suspect breast cancer
E. Blood pressure 130/80 mmHg

Question 128

A fetus does not have a fully developed immunologic system and needs to
receive help from the maternal system. Which immunoglobulins can cross
through the placenta to provide passive immunity for the fetus?

A. Only IgA
B. IgA and IgG
C. IgM and IgG
D. Only IgG
E. IgE and IgM
Question 129

An 18-year-old patient finally delivered a 4,000-g infant vaginally. Her


prenatal course was complicated by anemia, poor weight gain, and maternal
obesity. She also complained that the production of her breastfeeding milk is
not much. On abdominal palpation, the uterine fundus is soft and at the level
of umbilicus. Which one is true about the body system in a newborn?

A. the kidneys of the neonate can concentrate urine up to 3 times the


osmolality of the plasma, same as the adult
B. Ascorbic acid is stored in significant amount in the fetal tissues.
C. The ability of neonate to absorb fat is the sama to that of the older
children
D. The normal metabolic rate of the neonate in relation to body weight is
about the same of the adult.
E. The rate of fluid intake in the newborn is around sevent times as great in
relation to weight as in the adult

Question 130

Mrs Nana, a 28–year-old G2P1A0, her blood pressure is 160 /90 mmHg. Which
of the following is the appropriate drug and its regimen dosage ?

A. Hidrochlorothiazide tablet 10 mg, twice a day


B. Amlodipine tablet 10 mg, once a day
C. Atenolol tablet 10 mg , 3 times a day
D. Furosemid tablet 40 mg, once a day
E. Nifedipine sustained release 10 mg, 3 times a day

Question 131
A 20-year-old patient complained about painless mass on her labia located
on the 7 oclock direction. She has no fever. 1 year ago, she also has the same
complain and admitted to the hospital What is the risk factor of this disease?

A. Nutrition
B. Hygiene
C. Age
D. Genetics
E. Promiscuity

Question 132

In simple linear regression, which of the following is true if H0: β1 = 0 is


rejected?

A. Assuming that we do not commit a type I error, the relationship is linear


and sufficient strength to justify the use of sample regression equation
to predict and estimate Y for given value of X
B. Assuming that we do not commit a type I error, the relationship
between X and Y is not linear. Some curvilinear model provides a better
fit to the data
C. Assuming that we do commit a type I error, the relationship is linear and
sufficient strength to justify the use of sample regression equation to
predict and estimate Y for given value of X
D. Assuming that we do commit a type I error, there is a good fit of the
data to a linear model, but some curvilinear model might provide an
even better fit
E. Assuming that we do not commit a type I error, although the
relationship between X and Y may be linear it is not strong enough for X
to be of much value in predicting and estimating Y
Question 133

Describe the structure and function of the organs of the female internal
genitals In clinical practice, the term perineum is frequently used to describe
the area between the external genitalia and the anus. However, in anatomical
terms, the perineum is a diamond-shape structure.

A. line between ischial tubercles, sacrotuberous ligaments, and coccyx


B. line between ischial fossae, sacrotuberous ligaments, and coccyx
C. The anal triangle is bounded by …
D. line between ischial tuberosities, sacrotuberous ligaments, and coccyx
E. line between ischial spines, sacrotuberous ligaments, and coccyx

Question 134

At age 5 days, a baby become lethargic and refused to nurse. The baby
suddenly had seizures and the parents took the baby to the emergency room.
The triage nurse exclaimed: This baby looks like a pumpkin! While waiting to
be seen by a doctor, the baby became apneic and required intubation and
ventilation. Phenobarbital was administered, and 1.5 hours later the STB was
reported as 35 mg/dL. What is the correct possible diagnosis of the baby?

A. Pneumonia
B. Bilirubin encephalopathy bkn ini?
C. Epilepsy
D. Respiratory failure
E. Neonatal sepsis

Question 135

Mrs. Eva, a 35-year-old female, came to the emergency with chief complaint
acute onset pain on lower abdominal area. Physical examination found a
palpable mass at the right lower quadrant, with 10 cm of diameter. Bimanual
exam showed slightly enlarged 10x10x10 cm cystic mass, mobile, tenderness
(+), utery within normal limit. Urine test of pregnancy was negative. which is
most likely structure of pelvic adnexa affected?

A. cervix
B. appendix
C. ovaries
D. fallopian tube
E. uterus

Question 136

A 45-year-old woman went to primary health care presents with pain after
sexual intercourse with her husband. She also complains about foul smelled
vaginal discharge. Doctor advised her to do visual inspection with acetic as a
cervical cancer screening. Which of the following is eligible for the test?

A. A woman who did vaginal douching 3 hours prior


B. A woman who is diagnosed with Stage 2 cervical cancer
C. A woman who had sexual intercourse 5 days prior
D. A woman who is in contact with spermictide 24 hours prior
E. A woman who is having heavy menstrual bleeding

Question 137

Define the pelvic girdle and describe the bones and ligaments of the pelvis
Explain the three regions of the hip bone and identify their bony landmarks
Describe the openings of the pelvis and the boundaries of the greater and
lesser pelvis The pelvic girdle (hip girdle) is formed by a single bone, the hip
bone or coxal bone or coxae (coxal = hip), which serves as the attachment
point for each lower limb. Each hip bone, in turn, is firmly joined to the axial
skeleton. The right and left hip bones also converge anteriorly to attach to
each other. The hip bone, or coxal bone, forms the pelvic girdle portion of the
pelvis. The paired hip bones are the large, curved bones that form the lateral
and anterior aspects of the pelvis. When you place your hands on your waist,
you can feel the arching, superior margin of the ilium along your waistline.
This curved, superior margin of the ilium is the ....

A. iliac crest
B. iliac ramus
C. iliac tuberosity
D. iliac spine

Question 138

She also consult about the contraceptive method. As a doctor, your answer
and explanation based on your knowledge. Which is the following statement
appopriate for her consultion : Reducing breastfeeding duration or frequency
and introducing supplementary feeds are all associated with earlier return of
ovulation

A. Suckling activating the pulsatile release of thyroid hormone


B. The Lactational Amenorrhea Method of contraception (LAM) leading to
depression of uterine activity
C. Suckling activating the pulsatile release of gonadotropin-releasing
hormon
D. There is no direct relationship between the intensity of breastfeeding
and the length of time before ovulation resumes
E. Low levels of estrogen and progensteron affecting myoepithelial cell in
alveolus
Question 139

Describe the structure and function of the organs of the female internal
genitals The mons pubis is a pad of fat that is located at the anterior, over the
pubic bone. After puberty, it becomes covered in pubic hair. The labia majora
(labia = lips; majora = larger) are folds of hair-covered skin that begin just
posterior to the mons pubis. The thinner and more pigmented labia minora
(labia = lips; minora = smaller) extend medial to the labia majora.

The external female reproductive structures are referred to collectively as the


A. vagina
B. vulva
C. clitoris
D. perineum

Question 140

A 6-year-old girl is seen for a 10 day history of intense vulvar itching leading
to excoriation, the vulva are noted to be diffusely inflamed. What is the most
probable cause of these symptoms?

A. Enterobious vermicularis
B. Candida albicans infection
C. Atophic vulvitis
D. Sexual assault
E. Foreign body

Question 141
Which of the following is the most appropriate terms for the pelvic type with
small posterior sagittal diameter, convergent sidewalls, prominent ischial
spines, and narrow pubic arch?

A. Platypelloid.
B. Android.
C. Anthropoid.
D. Gynecoid.
E. Mixed.

Question 142

An experimental birth control method for women uses a chemical to directly


prevent pregnancy by altering the protein coating on sperm. The changed
protein coating on the sperm is likely to disrupt. Which one of the following is
the biological process of it?

A. Oogenesis
B. Spermatocytogenesis
C. Fertilization
D. Spermatogenesis
E. Ovulation

Question 143

A newborn girl, born at 35 weeks gestation, is brought to the transitional


nursery at 1 hour of life after the postpartum nurse expressed concerns about
her breathing. The mother is a 27-year-old primigravid woman with no
significant past medical history. Vaginal delivery occurred 20 hours after
rupture of membranes, and the Apgar scores were 8 and 9 at 1 and 5 minutes,
respectively. The baby is pink and active without respiratory support. Her vital
signs are normal except for a respiratory rate of 85 breaths per minute. The
physical examination is significant for mild nasal flaring, intercostal and
subcostal retractions, good bilateral air entry, and expiratory grunting. The
Spo2 is 98%. What is the risk factor for neonatal sepsis in this patient?

A. vaginal delivery
B. Prolonged rupture of membranes (20 hours)
C. Female
D. primigravida
E. Chorioamnionitis

Question 144

Describe the structure and function of the organs of the female internal
genitals In clinical practice, the term perineum is frequently used to describe
the area between the external genitalia and the anus. However, in anatomical
terms, the perineum is a diamond-shape structure. Where does fertilization of
the egg by the sperm typically occur?

A. ovaries
B. vagina
C. corpus luteum
D. oviducts

Question 145

A 2-day-old baby boy was brought to the hospital complaining of yellow skin
all over his body. What is the most appropriate patological process related to
the case?

A. Juvenille Gallstones
B. Hemolytic Uremic Syndrome
C. ABO Incompatibility
D. Idopathic Thrombocytopenic Purpura
E. Thrombotic Thrombocytopenia Purpura

Question 146

1. High levels of estradiol


2. High levels of estradiol and progesterone
3. Luteinizing Hormone (LH) surge
4. Menstruation
5. Presence of a corpus albicans Using the above list of events, indicate the
proper sequence concerning the ovarian/uterine cycle.

A. 1, 2, 3, 5, 4
B. 4, 2, 3, 1, 5
C. 1, 2, 3, 4, 5
D. 4, 2, 3, 5, 1
E. 1, 3, 2, 5, 4

Question 147

The pudendal nerve is formed from the sacral plexus. After re-entering the
pelvis, it accompanies the internal pudendal artery and vein, coursing
anterosuperiorly through pudendal canal. Pudendal canal is a structure
formed by ..

A. of the obturator externus muscle


B. of obdurator muscle
C. of ischiococcygeus muscle
D. of piriformis muscle
E. of the obturator internus muscle
Question 148

A 30-year-old woman came to the Dermatovenereology Clinic, present with


discharge since 1 week. Her last sexual intercourse is 2 weeks before came to
clinic with her boyfriend. Venereological examination showed yellowish
discharge. Which of the following information is the most important that can
educate patient for increasing awareness these diseases?

A. The patient can treat these diseases by themselves


B. These diseases have several complications that can impact patients
life
C. The disease can resolve itself
D. Condom can be used to prevent these disease
E. These diseases cant be cured

Question 149

Which of the following is an absolute contraindication to external cephalic


version?

A. Fetal-growth restrict
B. breech presentation
C. Oligohydramnios Respiratory distress syndrome
D. Early labor
E. Twin gestation

Question 150

A 19–year-old woman, G1P0A0 with 28 weeks gestational age, she is suffer


form respiratory infection , caused by Gram (+) bacteria. What is the most
correct antibiotic for her infection ?
A. Cotrimoxazole 1 dd 960 mg
B. Ciprofloxacin 1 d.d 500 mg
C. Cephadroxil 2 d.d 200 mg
D. .Penicillin V 3 d.d 1000 mg
E. Erythromycin 3 d.d. 500 mg → bukaan ini?

Question 151

A 15-year-old boy was involved in a traffic accident and was taken to the
hospital. The patient requires a blood transfusion. Blood group revealed B Rh+
blood group. What is the blood group that acts as recipient donor

A. O
B. D
C. .B
D. AB
E. A

Question 152
A male baby was born from a 20-year-old primigravida mother by vaginal
delivery with head presentation. Birthweight was 3700gr. The delivery was
rather difficult, as the mother could not push properly. On examination you
notice the head was bulging and on palpation you can feel the bulge extend
across sutures. What is the most possible diagnosis of your finding?

A. Subdural haemorrhage
B. Epidural hemorrhage
C. Subgaleal hematoma
D. Cephal hematoma
E. Caput succedaneum

Question 153

A baby with AB blood group was born at term gestation. Her mother has
blood group O, and Rh negative. On admission to the nursery, the nurses
thought that the babys skin had a yellow tinge. The physician believed this
was only very mild jaundice and chose to ignore it. An astute nurse, however,
took an STB at age 12 hours, the result of which was 9.2 mg/dL. Not very high,
responded by the physician. By the next day (28 hours) the STB value was 15
mg/dL. Which one of the following answer is correct about the physiology of
bilirubin?

A. Biliverdin reductase will reduce central methylene bridge from methyl


group into biliverdin
B. Bilirubin thats loosely tied hard to fasten & diffuse into tissues
C. Bilirubin is removed from albumin & will not be taken up
D. Bilirubin bound to serum albumin is more insoluble
E. ;Catabolism of all heme proteins happen in microsomal fraction by
heme oxygenase → ini jwbn cava
Question 154

A 2-day-old baby boy was brought to the hospital complaining of yellow skin
all over his body. Blood laboratory examination revealed an increase in
indirect bilirubin. What is the most appropriate underlying cause related to
the case?

A. Increase destruction of RBC ini gaa?


B. Destruction of bile acid
C. Obstruction of bile
D. Abnormalities of hepatic enzyme apa ini
E. Increase desctruction of hepatic cell

Question 155

A 34-year-old woman with vaginal bleeding at 30 weeks gestation. Bleeding


started 2 days ago and was initially spotting but now increased so she need
to change sanitary towel regularly. There is mild dull lower abdominal pain.

She is gravida 3 para 2. She used contraceptive pills but stopped 3 years ago.
There is no gynaecological history. She is fit medically. The abdomen non
distended but tender suprapubically. The cervical os is closed and products
conception still inside.

What will you do to this patient?

A. give tranexamic acid stop the bleeding


B. give antibiotic and corticosteroid treatment
C. give tocolytics such as nifedipine to delay early labour
D. do ultrasound examination to further explore the posibility of
miscarriage
E. no need further examination because its confirmed threatened labour
Question 156

One medical student wants to test a hypothesis that women with higher
childhood socioeconomic status (SES) have earlier age at menarche.
Suppose she identifies 40 girls with low childhood SES and 30 girls with high
childhood SES and observes mean age at menarche for girls with low and
high childhood SES are 13.4 years with standard deviation of 1.4 years and 12.9
years with standard deviation of 1.5 years, respectively. It is assumed that the
variances of the two groups are equal. If the assumptions for parametric test
are not met, which of the following is the correct alternative test?

A. Wilcoxon sum-rank test


B. Friedman test
C. Kruskal – Wallis test
D. Kolgomorov – Smirnov test
E. Wilcoxon signed-rank test

Question 157

A 21-day-old male infant was born at 30 weeks gestation with a birth weight
of 1200 grams. His history is notable for respiratory distress syndrome, he was
treated with surfactant. He remains intubated on conventional ventilator
support. Caffeine therapy was initiated. He received a 7-day course of
ampicillin and cefotaxime for presumed sepsis at birth and another 5-day
course of cefotaxime at 2 weeks of age. The infant continues to deteriorate
over the next 48 hours. His new CBC is notable for a white blood cell count of
5000/mL and a platelet count of 32,000/mL. Which of the following is not a
known risk factor for late onset sepsis in this infant?

A. Exposure to third-generation cephalosporins


B. Presence of an endotracheal tube
C. Presence of a central venous catheter
D. Caffeine therapy
E. Extremely low birth weight

Question 158

Bob and Miriam Pataki walk in to your office with their 16 year-old daughter
Nina, who has not yet menstruated for the first time. Examination revealed a
thin girl with normal breast development and the normal presence of pubic
and axillary hair. On talking with Nina, you discover she is a junior in high
school and is very active in sports. She is the highschools track and field star,
and she is an avid swimmer. Because of her track practice, Nina stays up late
most nights to do her homework, consequently, she only gets five to six hours
of sleep a night. What is the improper secretion of which of the following
hormones do you suspect to be the original culprit of Ninas amenorrhea?

A. Estradiol
B. luteinizing hormone (LH)
C. gonadotrophin releasing Hormone ( GnRH)
D. folicle stimulating hormone (FSH)

Question 159

Suppose a new treatment for pain relief is to be tested among patients with
disease X. The measure of pain relief will be the percent change in pain level
as reported by the patient after taking the medication for 1 month. What do
type I error mean in this situation?

A. To conclude that the treatment doesnt have an effect when in fact it


doesnt have an effect
B. To conclude that the treatment has an effect when in fact it has an
effect
C. To conclude that the treatment has an effect when in fact it doesnt
have an effect
D. To conclude that the treatment has an effect using inappropriate
statistical method
E. To conclude that the treatment doesnt have an effect when in fact it
has an effect

Question 160

Describe the pelvic vasculature: artery and vein

Describe the pelvic innervation The pudendal nerve is formed from the sacral
plexus. After its formation, pudendal nerve descends and leaves the pelvis
through the lower part of greater sciatic foramen and then re-enters the
pelvis through the lesser sciatic foramen.

A. sacroiliac ligament
B. sacrococcyx ligament
C. sacropubic ligament
D. sacrospinous ligament
E. Before re-entering the pelvis, pudendal nerve crosses the …

Question 161

A 34-year-old woman is brought to hospital due to post partum bleeding.


Which of the following is the drug of choice for this case?

A. Mefenamic acid
B. Ritordin
C. MgSO4
D. Methyl ergonovin
E. Misoprostol

Question 162

How long does complete uterine involution take following delivery?

A. 2 weeks
B. 3 weeks
C. ;4 weeks
D. 5 weeks
E. 1 week

Question 163

One medical student wants to test a hypothesis that women with higher
childhood socioeconomic status (SES) have earlier age at menarche.
Suppose she identifies 40 girls with low childhood SES and 30 girls with high
childhood SES and observes mean age at menarche for girls with low and
high childhood SES are 13.4 years with standard deviation of 1.4 years and 12.9
years with standard deviation of 1.5 years, respectively. It is assumed that the
variances of the two groups are equal. What is the value of the test statistic?

A. 1.45
B. 1.43
C. 1.42
D. 1.41
E. 1.44

Question 164

You are a doctor in a primary health facility. You have a case where there is a
baby girl, 20 hours of experiencing jaundice with yellow symptoms that have
reached the palms of the hands and soles of the feet. Which action or
treatment is right for the baby?

A. Breastfeed the baby more often


B. Exchange transfusion immediately, no need to wait for bilirubin serum
result
C. The baby does not need any intervention
D. It is a normal/physiological condition, continue with sunbathing therapy
E. Directly phototherapy while waiting for bilirubin serum result and
preapare for exchange transfusion

Question 165

Mrs Rara, a 25-year-old G1P0A0, with 10 weeks gestational age, present at PKM
due to nausea-vomiting every morning. She works as a teller of a private
bank. Which is the most appropriate antiemetic for her ?

A. Thiamine 25 mg
B. Diphenhydramine 25 mg
C. Doxylamin 25 mg
D. Meziklin 25 mg
E. Diphenhidrinate 50mg

Question 166

One medical student wants test a hypothesis that there is a correlation


between variable X and Y. To test the hypothesis H0: r = 0 versus H1: r ≠ 0, with
significance level α, which of the following is the correct decision rule to reject
H0 using t test?
A. t > tn – 3, 1 – α or t < -tn – 3, 1 – α
B. t > tn – 2, 1 – α/2 or t < -tn – 2, 1 – α/2
C. t > tn – 1, 1 – α/2 or t < -tn – 1, 1 – α/2
D. t > tn – 1, 1 – α or t < -tn – 1, 1 – α
E. t > tn – 2, 1 – α or t < -tn – 2, 1 – α

Question 167

Mrs Popi, a 21-year-old G1P0A0, with 12 weeks gestational age, present at


Gynecology policlinic due to headache and myalgia. Which of the following is
the best drug for her symptom ?

A. Tramadol
B. Ibuprofen
C. Prednisone
D. Paracetamol
E. Diclofenac

Question 168

A 45-year-old pregnant woman come to the obstetrician to do prenatal care


examination, she had a history of labour with spina bifida fetus. The doctor
suggests her to do amniocentesis examination. When amniocentesis could
be examined?

A. After 24 weeks of gestational age


B. After 12 weeks of gestational age
C. After 17 weeks of gestational age
D. After 20 weeks of gestational age
E. After 16 weeks of gestational age Hrsnya 15-20w
Question 169

Which of the following describes the function of hormone B in females?

A. Stimulate estrogen secretion from ovaries


B. Increase basal metabolic rate
C. Promote milk production
D. Stimulates steroid release from the ovaries
E. Promote growth of immature follicles in ovaries

Question 170

A 31 years old woman felt 31 weeks of pregnancy, come to the emergency


room with chief complaint of labour pain that increase in frequency and
intensity since 3 hours before admission.

This is her second pregnancy, and the patient regularly checks her pregnancy
to the midwife. Two weeks ago, at the last examination, revealed that the
baby was in breech position with 29 weeks of pregnancy. From the physical
examination :
- General appearance in good condition, mild ill, moderate pain when labor
pain felt
- Vital sign and general status within normal limit
Obstetric examination :
- Fundal height is 30 cm
- The baby in breech position, back on the left
- Fetal heart rate 148-152 bpm
- Uterine contraction 2x/10 minutes, each contraction sustain for 30 second,
strong intensity
- Estimated birth weight is 1900 grams
From the vaginal examination, the buttock is palpable without palpable
babys feet, the baby is in
A. Compound presentation
B. Kneeling presentation
C. Footling-frank breech presentation
D. Frank breech presentation
E. Footling presentation

Question 171

A multiparous woman in 9th month pregnancy come to the emergency ward

with complaint of painful contraction. Which of the following is indicate an

active labor

A. Progressive cervical dilatation and effacement


B. 4 cm dilatation of cervix
C. 3 contractions per 10 min
D. Painful contractions every 2-3 min
E. Painless, regular uterine contraction

Question 172

An experimental birth control method for women uses a chemical to directly


prevent pregnancy by altering the protein coating on sperm. The aim of this
method is likely to disrupt which of the following biological processes?

A. Oogenesis
B. Fertilization
C. Spermatogenesis
D. Spermatocytogenesis
E. Ovulation

Question 173
A 10-day-old newborn need red cell transfusion for massive bleeding due to
Vit.K deficiency. The result of blood grouping test showed: Anti-A: Negative,
Anti-B: Negative, Anti-AB: Negative, Anti-Rh: Positive What is the examination
for pretransfusion testing related to the case above?

A. Crossmatch test
B. Indole-test
C. LE test
D. Coombs test
E. HAMs Test

Question 174

A 19-year-old woman presented to the clinic with the primary complaint of


vaginal discharge that had been present since the previous week. She
confessed to being a promiscuous woman. The doctor found a frothy cream
discharge under the speculum and a motile protozoan with whip-like tail and
with jerky, twisting locomotion activity . Which one the pathogen is cause of
the complaint?

A. Toxoplasma gondii
B. Trichomonas vaginalis
C. Gardenalla vagnalis
D. Proteus mirabilis
E. Candida albican

Question 175

A 20-year-old patient complained about transparent vaginal discharge since


1 month ago. She did not complained about itchiness, foul smell, nor fever
What is the most likely diagnosis?
A. Gonorrahe
B. Chlamydia
C. Physiological Discharge
D. Bacterial vaginosis
E. Cervical cancer

Question 176

A 10-day-old newborn need red cell transfusion for massive bleeding due to
Vit.K deficiency. The result of blood grouping test showed: Anti-A: Negative,
Anti-B: Negative, Anti-AB: Negative, Anti-Rh: Positive What coagulant factor
mostly affected related to the case?

A. Factor II
B. Factor XII
C. Factor XIII
D. Factor III
E. Factor VIII

Question 177

The investigators want to compare cholesterol level of the children whose


fathers have died from heart disease versus the children whose fathers do
not have a history of heart disease. Suppose from 100 children whose fathers
have died from heart disease, the observed mean and standard deviation of
the cholesterol level are 207.3 mg/dl and 35.6 mg/dl. Another 80 children
whose fathers do not have a history of heart disease are also identified and
the observed mean and standard deviation of the cholesterol level are 193.4
mg/dl and 17.3 mg/dl. It is assumed that the variances of the two groups are
not equal. What is the value of the test statistic?
A. 3.60
B. 3.20
C. 3.30
D. 3.43
E. 3.53

Question 178

In simple linear regression, to test the hypothesis H0: β1 = 0 versus H1: β1 ≠ 0,


with significance level α, which of the following is the correct decision rule to
reject H0 using t test?

A. t > tn – 3, 1 – α or t < -tn – 3, 1 – α


B. t > tn – 1, 1 – α or t < -tn – 1, 1 – α
C. t > tn – 1, 1 – α/2 or t < -tn – 1, 1 – α/2
D. t > tn – 2, 1 – α/2 or t < -tn – 2, 1 – α/2
E. t > tn – 2, 1 – α or t < -tn – 2, 1 – α

Question 179

Mrs C, a 30-year-old, came to a clinic with chief complaint of heavy and


yellow-colored vaginal discharge. She claimed that she has no other
symptoms and there was no significant changes in her menstrual cycle
Vaginal discharge secretion will increase in the following event:

A. Use of IUD contraception


B. Fungal infection
C. Endometrial cancer
D. ;Ovulation
E. Puerperium
Question 180

A 24-year-old G1P0A0 patient came to primary health facility to do annual


antenatal care check up. She is prescribed folic acid tablets and ferrous
tablets by the doctor What is the recommended daily dosage of folic acid in
pregnant patient?

A. 1 mg
B. 600 mcg
C. 800 mcg
D. 100 mcg
E. 6 mg

Question 181

A newborn healthy baby girl, born at 36 weeks gestation. Her mother is a


27-year-old primigravid with no significant past medical history. The baby
delivered vaginally and was occurred for 10 hours. Which one is the correct
answer about cardiovascular changes in the new born baby?

A. ;The blood volume of a neonate immediately after birth averages about


375 milliliters.
B. Occasionally a child is born with an especially low cardiac output.
C. The cardiac output of the neonate averages 250 ml/min.
D. The heart rate varies from 80–100 per minute.
E. The arterial pressure during the first day after birth averages about 90
mmHg systolic and 60 mm Hg diastolic.

Question 182
A 40-year-old woman just delivered a week ago to a healthy child. She
delivered spontaneously. She has a bloody discharge. She need to change
tampon 5 times a day. She doesnt feel any pain just weak. What should you
do?

A. give tranexamic acid


B. refer to specialist for examination
C. give blood transfusion
D. explore the cause of bleeding
E. its normal so dont need further examination

Question 183

A 20-year-old patient complained about foul smelled vaginal discharge


since 1 month ago. She works as a sex worker in one of red district in the city
and has a history of promiscuity. Vaginal discharge is tested and found gram
negative bacteria with intra celullar body. Where is specimen of the discharge
most likely taken?

A. Anal
B. Posterior Fornix
C. Labia
D. Endocervix
E. Ectocervix

Question 184

Mother of a 5 year-old-boy visits your clinic and seeks for consultation


regarding her sons nutritional status. The child seems healthy; however he is
looking a little bit bigger for his age. If the body mass index measurement of
the child is between 85 and 95 percentiles of CDC 2000 curve. What would
you inform the parents regarding their child nutritional status?

A. Risk of overweight
B. Risk of obesity
C. Well nourished
D. Obesity
E. Overweight

Question 185

The diagnostic puncture may be followed by sonographically guided


drainage or in selected cases by therapeutic puncture, in the early pregnancy
< 12 weeks, biopsy could be performed savely to know the genetic
abnormality of the fetus. What is the most likely diagnosis technique of this
procedure ?

A. Cystoscopy guided Ultrasound


B. Chorionic villus sampling (CVS)
C. Fetal Blood Sampling (FBS)
D. Amniocentesis
E. CT Scan

Question 186

A 20-year-old pregnant woman with 32 weeks of gestation comes to clinic


with watery discharge since 2 hours ago. The woman was diagnosed to have
a preterm premature rupture of the membrane (PPROM). Which of the
following organism is most likely causing the disease of this patient and
cause the neonatal sepsis?
A. Enterococcus sp.
B. Group B Streptococcus
C. Staphylococcus saprophyticus
D. Staphylococcus epidermidis
E. Escherichia coli

Question 187

A mother and her 16-years old daughter present to your office because the
daughter has not yet menstruated. They are very concerned that something
is wrong. By applying principles of puberty to this patient, it is possible to
determine if the teen is simply undergoing a slightly delayed puberty versus
potentially manifesting a significant endocrine or anatomical problem. Which
of the following sequence of conditions should be happen in this patient if she
is normal?

A. thelarche, menarche, maximal growth velocity


B. maximal growth velocity, thelarche, menarche
C. menarche, maximal growth velocity, thelarche
D. maximal growth velocity, menarche, thelarche
E. thelarche, maximal growth velocity, menarche

Question 188

A woman 27 years old at 20-22 weeks gestation. She had had regular cycles
for 5 days every 28 days. She had no irregular bleeding or any abdominal
pain. She is on her third pregnancy. All her previous children delivered
spontaneously. If you do physical examination, where would you feel the
fundus at?

A. Above pubic bone


B. Umbilical
C. Below umbilical
D. Under the sternum
E. Above umbilical

Question 189

Describe the pelvic vasculature: artery and vein

Describe the pelvic innervation The pudendal nerve is formed from the sacral
plexus. After its formation, pudendal nerve descends and leaves the pelvis
through the lower part of greater sciatic foramen and then re-enters the
pelvis through the lesser sciatic foramen.

A. inferior rectal nerve, perineal nerve, ventral nerve of the penis or clitoris
B. The pudendal nerve supplies sensation to external genitals of both
sexes and the skin around the anus, anal canal, and perineum through
it branches:
C. inferior rectal nerve, perineal nerve, dorsal nerve of the penis or
clitoris
D. superior rectal nerve, perineal nerve, caudal nerve of the penis or clitoris
E. superior rectal nerve, perineal nerve, ventral nerve of the penis or clitoris

Question 190

The cervix is the least mobile part of the uterus because of the passive
support provided by attached condensations of endopelvic (ligaments),
which may also contain smooth muscle what is the most correct explained?

A. Uterosacral ligaments pass inferiorly and slightly posteriorly from the


sides of the cervix to the middle of the sacrum
B. Endometrium, the inner mucous coat is firmly adhered to the underlying
myometrium in uterus
C. Mesometrium, the largest part of the broad ligament, inferior to the
mesosalpinx and mesovarium
D. Broad ligament of the uterus, a double layer of peritoneum (mesentery)
that extends from the sides of the uterus to the lateral walls and floor of
the pelvis
E. Cardinal (transverse cervical) ligaments extend from the supravaginal
cervix and lateral parts of the fornix of the vagina to the lateral walls of
the pelvis

Question 191

A 45-year-old woman went to primary health care complained pain after


sexual intercourse with her husband. She also complained about foul smelled
vaginal discharged. Doctor advised her to do papsmear as a cervical cancer
screening Which instrument should be used to perform the examination?

A. Hegar dilator
B. Loop electrode
C. Ayre spatula bkn ini? Antara ini sm e, kl ini di ambil di portio
D. Speculum
E. Cytobrush kl ini di endocervix

Question 192

Define the pelvic girdle and describe the bones and ligaments of the pelvis

Explain the three regions of the hip bone and identify their bony landmarks
Describe the openings of the pelvis and the boundaries of the greater and
lesser pelvis The pelvic girdle (hip girdle) is formed by a single bone, the hip
bone or coxal bone or coxae (coxal = hip), which serves as the attachment
point for each lower limb. Each hip bone, in turn, is firmly joined to the axial
skeleton. The right and left hip bones also converge anteriorly to attach to
each other.

The hip bone, or coxal bone, forms the pelvic girdle portion of the pelvis. The
paired hip bones are the large, curved bones that form the lateral and
anterior aspects of the pelvis.

The pubis forms the anterior portion of the hip bone. The enlarged medial
portion of the pubis is the pubic body. Located superiorly on the pubic body is
a small bump called the …

A. ischiopubic ramus
B. pubic tubercle
C. superior pubic ramus
D. pubic arch

Question 193

One medical student wants to test a hypothesis that there is a score


difference given from two examiners during SOOCA examination in Faculty of
Medicine, University Y (for one examination room only). If the assumptions for
parametric test are not met, which of the following is the correct alternative
test?

A. Kolmogorov – Smirnov test


B. Kruskal – Wallis test
C. Wilcoxon sum-rank test
D. Friedman test
E. ; Wilcoxon signed-rank test

Question 194

A 37-weeks gestation woman was admitted from clinic yesterday. This is her
second pregnancy. Her first child was born spontaneously 10 years ago. She
remmaried. Her blood pressure at 13 weeks was 139/75mmHg. Her blood tests
were unremarkable. 2 weeks ago her blood pressure 140/83 mmHg. Her
urinalysis result was normal. The blood pressure was repeated 2 days later
(140/85mmHg). Two days ago she did further appointment with blood
pressure 147/100 mmHg. Her legs was swollen. She denies any headache or
blurring vision.

A few minutes ago suddenly she has seizure. What is your interpretation and
further management needed?

A. preeclampsia, monitor fetal well being and induced labour


B. preeclampsia, induction of labour as soon as possible
C. preeclampsia, wait for spontaneous labor
D. eclampsia, elective caesarean section
E. eclampsia, monitor fetal and maternal well being, induced labor

Question 195

A woman just do MR imaging after her HSG results showed dibverging horns
on her uterine. It is confirmed she had bicornuate uterus. What are adverse
obstetrical outcomes below, EXCEPT:
A. preterm birth
B. malpresentation
C. malposition
D. microsomia
E. Misscarriage

Question 196

Define the pelvic girdle and describe the bones and ligaments of the pelvis

Explain the three regions of the hip bone and identify their bony landmarks

Describe the openings of the pelvis and the boundaries of the greater and
lesser pelvis The pelvic girdle (hip girdle) is formed by a single bone, the hip
bone or coxal bone or coxae (coxal = hip), which serves as the attachment
point for each lower limb. Each hip bone, in turn, is firmly joined to the axial
skeleton. The right and left hip bones also converge anteriorly to attach to
each other.

The hip bone, or coxal bone, forms the pelvic girdle portion of the pelvis. The
paired hip bones are the large, curved bones that form the lateral and
anterior aspects of the pelvis.

A. Each adult coxal bone is formed by …


B. ilium, pubic, coccyc
C. ilium, ischium, and pubic
D. ischium, sacrum, coccyc
E. ischium, sacrum, pubic

Question 197
A 22-year-old primigravida presents in active labor at 4 cm dilation. Her
membranes are intact, the fetus is cephalic, and the fetal tracing is category I.
Two hours later, she is still 4 cm dilated. Her contractions are every 7 minutes.
What is the best next step in labor management?

A. Oxytocin augmentation
B. Amniotomy and oxytocin
C. Continued observation
D. Cesarean section for failure to progress
E. Amniotomy

Question 198

A 28-year-old G2P1A0 at 12th week gestational age came to the doctor for her
first prenatal care. Based on the screening for TORCH the patient was
diagnosed with maternal acute toxoplasmosis. The USG result showed no
abnormality of the fetus. The patient was treated with oral Spiramycin. What is
the mechanism of action of the drug given to this patient?

A. Inhibit mitochondria synthesis


B. Inhibit cell wall synthesis
C. Inhibit folic alid synthesis
D. Protein inhibitor
E. Inhibit ribosome synthesis

Question 199

A 64-year-old woman presented with abdominal bloating, low back pain,


early satiety and progressive fatigue. She underwent hysterectomy 5 years
ago for benign indication; hypothyroidism managed medically; generalized
anxiety disorder managed medically. On physical examination, there was
found diffuse lumbosacral pain with movement; abdominal tenderness and
significant abdominal distension with a fluid weight consistent with ascites;
unintentional weight loss of 4 kgs. She was diagnosed with ovarian cancer.
Who is most at risk for developing ovarian cancer?

A. Woman who is underweight


B. Women with multiple children
C. Women over the age of 60
D. Woman with thyroid problems
E. Woman with history of breast cancer

Question 200

A 34-year-old P2A0 nursing mother came to Primary Health Care to consult


getting hormonal contraception. She refused to get an IUD. What is the
appropriate contraception for her?

A. Contraceptive implant
B. Combination progesterone and estrogen
C. Progestin only pills
D. Contraceptive pills with bi-phasic regimen
E. Estrogen only pills

fGausah diapus tkt ada yg belum beres

CRP → 40590

Question 1
An important hypothesis in hypertension research is that the sodium restriction may
lower blood pressure. However, it is difficult to achieve sodium restriction over long
term, and dietary counseling in a group settfing is sometimes used to achieve this goal.
The investigators measures the urinary sodium excretion (mEq/8hr) from 25 subjects at
baseline and after 1 week of dietary counseling. At baseline, the mean and the standard
deviation of the sodium excretion are 17.65 mEq/8hr and 10.56 mEq/8hr, respectively.
After one week, the mean and the standard deviation of the sodium excretion are 16.5
mEq/8hr and 11.63, respectively. The standard deviation of the difference is 12.22.
What is the value of the test statistic?

A. -0.77
B. -0.47
C. -0.67
D. -0.57
E. -0.37

Question 2
An important hypothesis in hypertension research is that the sodium restriction may
lower blood pressure. However, it is difficult to achieve sodium restriction over long
term, and dietary counseling in a group setting is sometimes used to achieve this goal.
The investigators measures the urinary sodium excretion (mEq/8hr) from 25 subjects at
baseline and after 1 week of dietary counseling. At baseline, the mean and the standard
deviation of the sodium excretion are 17.65 mEq/8hr and 10.56 mEq/8hr, respectively.
After one week, the mean and the standard deviation of the sodium excretion are 16.5
mEq/8hr and 11.63, respectively. The standard deviation of the difference is 12.22. If the
assumptions for parametric test are not met, which of the following is the correct
alternative test?

A. Wilcoxon signed-rank test


B. Friedman test
C. Kruskal – Wallis test
D. Kolgomorov – Smirnov test → sisanya parametric
E. Wilcoxon sum-rank test

Question 3
The Womens CARE study examined the use of OCs as a risk factor for breast cancer in
women ages 35 to 64. One of the findings described as follow: from women who were
diagnosed with breast cancer, 200 of them were current user and 1032 were non-user.
From women who did not have breast cancer, 172 of them were current user and 980
were non-user. Current use was defined as use of combination oral contraceptives
within six months preceding the reference date. What is the expected count for non-user
among case group?

A. 192.2
B. 1039.8 ini deh kan (2,012 x 1,232)/2,384
C. 2384.6
D. 972.2
E. 179.8

Question 4
The Womens CARE study examined the use of OCs as a risk factor for breast cancer in
women ages 35 to 64. One of the findings described as follow: from women who were
diagnosed with breast cancer, 200 of them were current user and 1032 were non-user.
From women who did not have breast cancer, 172 of them were current user and 980
were non-user. Current use was defined as use of combination oral contraceptives
within six months preceding the reference date. What is the value of the test statistic?

A. 0.78
B. 0.79
C. 0.45
D. 0.67
E. 0.56

Question 5
Because of increased rates of respiratory complications, elective cesarean delivery is
discouraged before 39 weeks of gestation unless there is evidence of fetal lung
maturity. Based on the background, the investigators assessed associations between
elective cesarean delivery at term (37 weeks of gestation or longer) but before 39
weeks of gestation and neonatal outcomes. Which of the following are the dependent
and independent variables for the study?

A. Dependent : fetal lung maturity


Independent : elective cesarean delivery at term
B. Dependent : neonatal outcomes
Independent : elective cesarean delivery at term iniiii
C. Dependent : elective cesarean delivery at term
Independent : fetal lung maturity
D. Dependent : elective cesarean delivery at term
Independent : neonatal outcomes kebalk
E. Dependent : before 39 weeks of gestation
Independent : elective cesarean delivery at term

Question 6
According to the Table 8, which of the following statements is appropriate?

A. There are ten qualitative data with nominal scale


B. There are thirteen qualitative data with nominal scale
C. There are fifteen qualitative data with nominal scale
D. There are twelve qualitative data with nominal scale
E. There are fourteen qualitative data with nominal scale
Question 7
According to the Table 15, which of the following statements is appropriate?

A. There are nine qualitative data, five with ordinal scale and four with nominal scale
B. There are nine qualitative data, seven with ordinal scale and two with nominal
scale
C. There are nine qualitative data, four with ordinal scale and five with nominal scale
D. There are four quantitative data and five qualitative data
E. There are nine qualitative data, six with ordinal scale and three with nominal
scale

Question 8
Suppose a new treatment for pain relief is to be tested among patients with disease X.
The measure of pain relief will be the percent change in pain level as reported by the
patient after taking the medication for 1 month. What do type II error mean in this
situation?

A. To conclude that the treatment has an effect when in fact it has an effect
B. To conclude that the treatment doesnt have an effect when in fact it has an effect
→ bener? iyaaa
C. To conclude that the treatment doesnt have an effect when in fact it doesnt have
an effect
D. To conclude that the treatment has an effect when in fact it doesnt have an effect
E. To conclude that the treatment has an effect using inappropriate statistical
method

Question 9
According to the Table 13, which of the following statements is appropriate?

A. There are ten qualitative data with nominal scale


B. There are eleven qualitative data with nominal scale
C. There are eight qualitative data with nominal scale
D. There are nine qualitative data with nominal scale
E. There are twelve qualitative data with nominal scale

Question 10
According to the Table above, which of the following statements is appropriate?

A. There are two quantitative data


B. There are two qualitative data with nominal scale
C. There is one qualitative data
D. There are three qualitative data with ordinal scale
E. There are two qualitative data with ordinal scale

uestion 11
Symptoms of gastroesophageal reflux are common in pregnancy, but there are limited
data on the risk of birth defects associated with exposure to proton-pump inhibitors
(PPIs) in early pregnancy. The investigators conducted a cohort study to assess the
association between exposure to PPIs during early pregnancy and the risk of major birth
defects among all infants born alive in Denmark between January 1996 and September
2008. Which of the following are the dependent and independent variables for the
study?

A. Dependent : major birth defects among all infants born alive


Independent : exposure to PPIs during early pregnancy
B. Dependent : exposure to PPIs during early pregnancy
Independent : gastroesophageal reflux in pregnancy
C. Dependent : major birth defects among all infants born alive
Independent : gastroesophageal reflux in pregnancy
D. Dependent : gastroesophageal reflux in pregnancy
Independent : exposure to PPIs during early pregnancy
E. Dependent : exposure to PPIs during early pregnancy
Independent : major birth defects among all infants born alive

Question 12
Suppose serum-cholesterol levels in spouse pairs (husband and wife) are measures to
determine whether there is correlation between cholesterol levels in spouses. Suppose
that the coefficient of correlation is 0.31 based on 51 spouse pairs. What is the value of
the test statistics?

A. 2.33
B. 2.26
C. 2.31
D. 2.28
E. 2.35
Question 13
The Womens CARE study examined the use of OCs as a risk factor for breast cancer in
women ages 35 to 64. One of the findings described as follow: from women who were
diagnosed with breast cancer, 200 of them were current user and 1032 were non-user.
From women who did not have breast cancer, 172 of them were current user and 980
were non-user. Current use was defined as use of combination oral contraceptives
within six months preceding the reference date. Which of the following is the proportion
of current user among control group?

A. 0.156
B. 0.162
C. 0.167
D. 0.149
E. 0.174

Question 14
According to the Table 7, which of the following statements is appropriate?

A. There are six qualitative data with ordinal scale


B. There are six qualitative data with nominal scale
C. There are four qualitative data with ordinal scale
D. There are four quantitative data
E. There are four qualitative data with nominal scale

Question 15
The Womens CARE study examined the use of OCs as a risk factor for breast cancer in
women ages 35 to 64. One of the findings described as follow: from women who were
diagnosed with breast cancer, 200 of them were current user and 1032 were non-user.
From women who did not have breast cancer, 172 of them were current user and 980
were non-user. Current use was defined as use of combination oral contraceptives
within six months preceding the reference date. Which of the following is the proportion
of current user among case group?
A. 0.162
B. 0.167
C. 0.174
D. 0.149
E. 0.156

Question 16
According to the Table 3, which of the following statements is appropriate?
A. There are five qualitative data with nominal scale
B. There are two quantitative data
C. There are two qualitative data with ordinal scale
D. There are seven qualitative data with nominal scale
E. There are five qualitative data with ordinal scale

Question 17
Childbearing is an established risk factor for urinary incontinence among young and
middle-aged women. It has been suggested that vaginal delivery is the main
contributing factor, possibly because of damage to important muscle tissue or nerves.
However, pregnancy itself may causes mechanical changes, hormonal changes, or both
that can lead to urinary incontinence. Results of epidemiologic and pathophysiological
studies assessing the relation between the mode of delivery and incontinence have
been inconclusive. A methodologic problem in many studies is the relatively small
number of participants, who are sometimes drawn from selected populations. The
investigators studied women in the Norwegian Epidemiology of Incontinence in the
County of Nord Trondelag (EPINCONT) study, a large community-based study, to
evaluate the risks of incontinence associated with cesarean section and vaginal
delivery. Which of the following are the dependent and independent variables for the
study?

A. Dependent : urinary incontinence


Independent : method of delivery iniiii ga sihhh stuju
B. Dependent : damage to important muscle tissue or nerves
Independent : method of delivery
C. Dependent : method of delivery
Independent : mechanical changes, hormonal changes, or both
D. Dependent : mechanical changes, hormonal changes, or both
Independent : method of delivery
E. Dependent : method of delivery
Independent : urinary incontinence Kebalik gasih

Question 18
The Womens CARE study examined the use of OCs as a risk factor for breast cancer in
women ages 35 to 64. One of the findings described as follow: from women who were
diagnosed with breast cancer, 200 of them were current user and 1032 were non-user.
From women who did not have breast cancer, 172 of them were current user and 980
were non-user. Current use was defined as use of combination oral contraceptives
within six months preceding the reference date. What is the expected count for current
user among case group?

A. 2384.6
B. 192.2
C. 1039.8
D. 972.2
E. 179.8

Question 19

According to the Table 12, which of the following statements is appropriate?

A. There are seven qualitative data with nominal scale


B. There are nine qualitative data with nominal scale
C. There are eight qualitative data with nominal scale
D. There are five qualitative data with nominal scale
E. There are six qualitative data with nominal scale

Question 20
The Womens CARE study examined the use of OCs as a risk factor for breast cancer in
women ages 35 to 64. One of the findings described as follow: from women who were
diagnosed with breast cancer, 200 of them were current user and 1032 were non-user.
From women who did not have breast cancer, 172 of them were current user and 980
were non-user. Current use was defined as use of combination oral contraceptives
within six months preceding the reference date. What is the expected count for current
user among control group?
A. 972.2
B. 1039.8
C. 2384.6
D. 179.8
E. 192.2

Question 21
According to the Table 10, which of the following statements is appropriate?
A. There are four quantitative data and four qualitative data with ordinal scale
B. There are three quantitative data and five qualitative data with nominal scale
C. There are five quantitative data and three qualitative data with ordinal scale
D. There are three quantitative data and five qualitative data with ordinal scale
E. There are four quantitative data and four qualitative data with nominal scale

Question 22
In simple linear regression, to test the hypothesis H0: β1 = 0 versus H1: β1 ≠ 0, with
significance level α, which of the following is the correct decision rule to reject H0 using t
test?

A. t > tn – 1, 1 – α/2 or t < -tn – 1, 1 – α/2


B. t > tn – 3, 1 – α or t < -tn – 3, 1 – α
C. t > tn – 1, 1 – α or t < -tn – 1, 1 – α
D. t > tn – 2, 1 – α/2 or t < -tn – 2, 1 – α/2
E. t > tn – 2, 1 – α or t < -tn – 2, 1 – α

Question 23
According to the Table 9, which of the following statements is appropriate?

A. There are fifteen quantitative data


B. There are thirteen quantitative data
C. There are fourteen quantitative data
D. There are eleven quantitative data
E. There are twelve quantitative data

Question 24
According to the Table above, which of the following statements is appropriate?
A. There are three qualitative data with ordinal scale
B. There are four qualitative data with ordinal scale
C. There are two qualitative data with nominal scale
D. There are two qualitative data with ordinal scale
E. There are three qualitative data with nominal scale

Question 25
The Womens CARE study examined the use of OCs as a risk factor for breast cancer in
women ages 35 to 64. One of the findings described as follow: from women who were
diagnosed with breast cancer, 200 of them were current user and 1032 were non-user.
From women who did not have breast cancer, 172 of them were current user and 980
were non-user. Current use was defined as use of combination oral contraceptives
within six months preceding the reference date. What is the point estimate and a 95%
CI for OR relating OC use and risk of breast cancer?

A. 1.05 (0.92; 1.21)


B. 1.05 (0.97; 1.15)
C. 1.10 (0.92; 1.33)
D. 1.05 (0.95; 1.17)
E. 1.10 (0.88; 1.38)

Question 26
One medical student wants to test a hypothesis that birthweight of an infant is
associated with smoking status of the mother during first semester of pregnancy. She
records birthweights of infants (grams) and smoking status of the mother and divides
the smoking status into four groups: non-smoker, ex-smoker, current smoker and
smokes less than 1 pack per day, and current smoker and smokes 1 pack per day or
more. If the assumptions for parametric test are not met, which of the following is the
correct alternative test?

A. Kolgomorov – Smirnov test


B. Friedman test
C. Wilcoxon signed-rank test
D. Kruskal – Wallis test
E. Wilcoxon sum-rank test

Question 27
According to the Table 14, which of the following statements is appropriate?

A. There are nine qualitative data, four with ordinal scale and five with nominal scale
B. There are nine qualitative data, three with ordinal scale and six with nominal
scale
C. There are nine qualitative data, six with ordinal scale and three with nominal
scale
D. There are nine qualitative data, seven with ordinal scale and two with nominal
scale
E. There are nine qualitative data, five with ordinal scale and four with nominal scale

Question 28
One medical student wants to test a hypothesis that women with higher childhood
socioeconomic status (SES) have earlier age at menarche. Suppose she identifies 40
girls with low childhood SES and 30 girls with high childhood SES and observes mean
age at menarche for girls with low and high childhood SES are 13.4 years with standard
deviation of 1.4 years and 12.9 years with standard deviation of 1.5 years, respectively.
It is assumed that the variances of the two groups are equal. What is the value of the
test statistic?

A. 1.43
B. 1.42
C. 1.44
D. 1.45
E. 1.41

Question 29
Which of the following is the correct property of correlation coefficient?

A. Its magnitude indicates how close the points are to the curve line
B. Its sign indicates the proportion of the variability of the relationship
C. Variable X and Y can be interchangeable without affecting its magnitude
D. Its value is valid only within the range of values of X and Y in the population
E. Correlation between x and y does necessarily imply a cause – effect relationship
Knp ga ini setuju iniii → harusnya ga ada sebab akibat (pembahasan cava)

Question 30
One medical student wants to test a hypothesis that there is a score difference given
from two examiners during SOOCA examination in Faculty of Medicine, University Y (for
one examination room only). If the assumptions for parametric test are not met, which of
the following is the correct alternative test?

A. Kolmogorov – Smirnov test


B. Wilcoxon signed-rank test
C. Kruskal – Wallis test
D. Friedman test
E. Wilcoxon sum-rank test

Question 31
A randomized trial was done to answer whether the knowledge of fetal oxygen
saturation, as an adjunct to electronic fetal monitoring, is associated with a change in
the rate of cesarean deliveries or the infants condition at birth. The investigators
randomly assigned 5341 nulliparous women who were at term and in early labor to
either open or masked fetal pulse oximetry. In the open group, fetal oxygen saturation
values were displayed to the clinician. In the masked group, the fetal oxygen sensor
was inserted and the values were recorded by computer, but the data were hidden.
Which of the following are the dependent and independent variables for the study?

A. Dependent : rate of cesarean deliveries fetal oxygen saturation


Independent : the infants condition at birth
B. Dependent : rate of cesarean deliveries
Independent : fetal oxygen saturation
C. Dependent : the infants condition at birth
Independent : fetal oxygen saturation
D. Dependent : the infants condition at birth rate of cesarean deliveries
Independent : fetal oxygen saturation ngga ini yaaa? akuini
E. Dependent : fetal oxygen saturation the infants condition at birth
Independent : rate of cesarean deliveries

Question 32
According to the Table 6, which of the following statements is appropriate?

A. There are three quantitative data


B. There are six qualitative data with nominal scale
C. There are five quantitative data
D. There are four qualitative data with nominal scale
E. There are two qualitative data with nominal scale

Question 33
Benign breast disease is an important risk factor for breast cancer. One historical cohort
study was conducted using large group of women with benign breast disease to obtain
reliable estimates of this risk. The conclusion was that risk factors for breast cancer
after the diagnosis of benign breast disease include the histologic classification of a
benign breast lesion and a family history of breast cancer. Suppose one medical student
wants to conduct a case control study to identify risk of breast cancer among women in
her region based on the previous study results. Which of the following are the
dependent and independent variables for her study?

A. Dependent : breast cancer


Independent : a family history of breast cancer
B. Dependent : breast cancer
Independent : histologic classification of a benign breast lesion
C. Dependent : breast cancer
Independent : large group of women
D. Dependent : breast cancer
Independent : family history of breast cancer
histologic classification of a benign breast lesion
E. Dependent : breast cancer
Independent : large group of women a family history of breast cancer

Question 34
The investigators want to compare cholesterol level of the children whose fathers have
died from heart disease versus the children whose fathers do not have a history of heart
disease. Suppose from 100 children whose fathers have died from heart disease, the
observed mean and standard deviation of the cholesterol level are 207.3 mg/dl and 35.6
mg/dl. Another 80 children whose fathers do not have a history of heart disease are also
identified and the observed mean and standard deviation of the cholesterol level are
193.4 mg/dl and 17.3 mg/dl. It is assumed that the variances of the two groups are not
equal. If the assumptions for parametric test are not met, which of the following is the
correct alternative test?

A. Friedman test
B. Wilcoxon signed-rank test
C. Kruskal – Wallis test
D. Wilcoxon sum-rank test
E. Kolmogorov – Smirnov test
Question 35
In simple linear regression, which of the following is true if H0: β1 = 0 is rejected?

A. Assuming that we do not commit a type I error, the relationship between X and Y
is not linear. Some curvilinear model provides a better fit to the data
B. ;Assuming that we do commit a type I error, the relationship is linear and
sufficient strength to justify the use of sample regression equation to predict and
estimate Y for given value of X
C. Assuming that we do not commit a type I error, the relationship is linear and
sufficient strength to justify the use of sample regression equation to predict and
estimate Y for given value of X
D. Assuming that we do not commit a type I error, although the relationship between
X and Y may be linear it is not strong enough for X to be of much value in
predicting and estimating Y
E. Assuming that we do commit a type I error, there is a good fit of the data to a
linear model, but some curvilinear model might provide an even better fit

Question 36
According to the Table 8, which of the following statements is appropriate?

A. There are eight quantitative data


B. There are seven quantitative data
C. There are four quantitative data
D. There are six quantitative datacare
E. There are five quantitative data

Question 37
The Womens CARE study examined the use of OCs as a risk factor for breast cancer in
women ages 35 to 64. One of the findings described as follow: from women who were
diagnosed with breast cancer, 200 of them were current user and 1032 were non-user.
From women who did not have breast cancer, 172 of them were current user and 980
were non-user. Current use was defined as use of combination oral contraceptives
within six months preceding the reference date. What is the expected count for non-user
among control group?

A. 2384.6
B. 1039.8
C. 179.8
D. 972.2
E. 192.2

Question 38
The investigators want to compare cholesterol level of the children whose fathers have
died from heart disease versus the children whose fathers do not have a history of heart
disease. Suppose from 100 children whose fathers have died from heart disease, the
observed mean and standard deviation of the cholesterol level are 207.3 mg/dl and 35.6
mg/dl. Another 80 children whose fathers do not have a history of heart disease are also
identified and the observed mean and standard deviation of the cholesterol level are
193.4 mg/dl and 17.3 mg/dl. It is assumed that the variances of the two groups are not
equal. What is the value of the test statistic?

A. 3.2
B. 3.43
C. 3.53
D. 3.6
E. 3.3

Question 39

According to the Table 5, which of the following statements is appropriate?


A. There are five qualitative data with nominal scale
B. There are two qualitative data with nominal scale
C. There are three qualitative data with nominal scale
D. There are six qualitative data with nominal scale
E. There are four qualitative data with nominal scale

Question 40
Suppose the investigator want to compare fasting serum-cholesterol levels among
recent immigrants to country A with typical levels found in the general population of
country X. Suppose it is assumed that the cholesterol levels in general population are
approximately normally distributed with mean 190 mg/dl. Blood test are performed on
100 immigrants, and it is observed that the mean is 181.52 mg/dl, with standard
deviation = 30 mg/dl. What is the value of the test statistic?
A. -2.63
B. -2.83
C. -2.73
D. -2.53
E. -2.43

Question 41
According to the Table 13, which of the following statements is appropriate?
A. There are five quantitative data
B. There are two quantitative data
C. There are four quantitative data
D. There are six quantitative data
E. There are three quantitative data

uestion 42
According to the Table 4, which of the following statements is appropriate?
A. There are thirteen quantitative data
B. There are eleven quantitative data
C. There are twelve quantitative data
D. There are fourteen quantitative data
E. There are fifteen quantitative data

Question 43
In simple linear regression, to test the hypothesis H0: β1 = 0 versus H1: β1 ≠ 0, with
significance level α, which of the following is the correct decision rule to reject H0 using
F test?

A. F > F1, n – 5, 1 – α
B. F > F1, n – 1, 1 – α
C. F > F1, n – 3, 1 – α
D. F > F1, n – 2, 1 – α
E. F > F1, n – 4, 1 – α

Question 44
According to the Table 11, which of the following statements is appropriate?

A. There are eight quantitative data


B. There are ten quantitative data
C. There are six quantitative data
D. There are seven quantitative data
E. There are nine quantitative data

Question 45
One medical student wants test a hypothesis that there is a correlation between variable
X and Y. To test the hypothesis H0: ρ = 0 versus H1: ρ ≠ 0, with significance level α,
which of the following is the correct decision rule to reject H0 using t test?

A. t > tn – 2, 1 – α/2 or t < -tn – 2, 1 – α/2


B. t > tn – 2, 1 – α or t < -tn – 2, 1 – α
C. t > tn – 3, 1 – α or t < -tn – 3, 1 – α
D. t > tn – 1, 1 – α/2 or t < -tn – 1, 1 – α/2
E. t > tn – 1, 1 – α or t < -tn – 1, 1 – α

Question 46
In simple linear regression, which of the following is true if H0: β1 = 0 is not rejected?

A. Assuming that we have made a type II error, although the relationship between X
and Y may be linear it is not strong enough for X to be of much value in
predicting and estimating Y
B. Assuming that we have not made a type II error, the relationship between X and
Y is not linear. Some curvilinear model provides a better fit to the data
C. Assuming that we have not made a type II error, There is a good fit of the data to
a linear model, but some curvilinear model might provide an even better fit
D. Assuming that we have not made a type II error, The relationship is linear and
sufficient strength to justify the use of sample regression equation to predict and
estimate Y for given value of X
E. Assuming that we have made a type II error, the relationship between X and Y is
not linear. Some curvilinear model provides a better fit to the data

Question 47
One medical student wants to test a hypothesis that women with higher childhood
socioeconomic status (SES) have earlier age at menarche. Suppose she identifies 40
girls with low childhood SES and 30 girls with high childhood SES and observes mean
age at menarche for girls with low and high childhood SES are 13.4 years with standard
deviation of 1.4 years and 12.9 years with standard deviation of 1.5 years, respectively.
It is assumed that the variances of the two groups are equal. If the assumptions for
parametric test are not met, which of the following is the correct alternative test?

A. Wilcoxon signed-rank test


B. Wilcoxon sum-rank test
C. Kruskal – Wallis test
D. Kolgomorov – Smirnov test
E. Friedman test

Question 48
Suppose a new treatment for pain relief is to be tested among patients with disease X.
The measure of pain relief will be the percent change in pain level as reported by the
patient after taking the medication for 1 month. What do type I error mean in this
situation?

A. To conclude that the treatment has an effect when in fact it has an effect
B. To conclude that the treatment has an effect when in fact it doesnt have an effect
C. To conclude that the treatment has an effect using inappropriate statistical
method
D. To conclude that the treatment doesnt have an effect when in fact it has an effect
E. To conclude that the treatment doesnt have an effect when in fact it doesnt have
an effect

Question 49
According to the Table 4, which of the following statements is appropriate?

A. There are two qualitative data with nominal scale


B. There are four qualitative data with nominal scale
C. There are six qualitative data with nominal scale
D. There are five qualitative data with nominal scale
E. There are three qualitative data with nominal scale

BHP
40623

Case 1
● Ibu A, 35 tahun adalah ibu dari empat orang anak perempuan.
Anak paling kecil berusia tiga tahun. Dia mengalami anemia
ringan, mengatakan dia sering lelah. Dia tinggal bersama
suaminya di rumah yang sederhana di pedesaan. Mereka hidup
dari bercocok tanam, memberi makan beberapa hewan
peliharaan, mengumpulkan kayu bakar, dan membawa beberapa
produk ke pasar local di kota terdekat.
● Dia datang ke klinik Keluarga Berencana terdekat dan meminta
dokter D untuk mensterilkannya karena dia merasa dengan
alasan kesehatan dan kemampuan keluarga yang terbatas, dia
tidak dapat mengatasi kehamilan serta membesarkan anak lagi.
● Dia mengatakan bisa membayar prosedur tindakan medis dari
tabungannya tetapi dia meminta dokter D dan staf klinik tidak
akan memberi tahu suaminya karena dia bisa melakukan
kekerasan dan selalu ingin menjadi ayah seorang anak laki-laki.
Dalam budaya lokal, para suami mengharapkan untuk
dikonsultasikan tentang perawatan medis istri mereka, tetapi hal
ini tidak diwajibkan secara hukum.

1.Apa implikasi etis dari kasus ini?


→ Autonomy, yaitu suatu hak seseorang untuk mendapatkan
perlakuan kepada tubuhnya sendiri. Pengaplikasian aspek autonomi
yaitu menghargai keputusan pasien untuk sterilisasi dan menjaga
kerahasiaan dari suaminya dikarenakan suaminya sering melakukan
KDRT. Sebelum membuat keputusan ini, dokter harus
mempertimbangkan risiko dan keuntungan dari tindakan yang
dilakukan baik secara medis maupun aspek budaya dan norma
setempat.
→ Beneficence: Dalam kasus ini, keuntungan yang didapatkan oleh
pasien lebih besar mengingat kondisi kesehatan ibu yang sering lelah
dan anemia, juga sosioekonomi pasien yang terbilang kurang.
→ Non-maleficence, yaitu tidak merugikan pasien. Dokter perlu
melakukan tindakan yang tidak membahayakan pasien yang tidak
bertentangan secara regulasi, norma, dan budaya. Dokter terlebih
dahulu juga harus menjelaskan ke pasien secara jelas bahwa
tindakan yang dilakukan tetap akan menimbulkan luka yang berisiko
ketauan oleh suaminya dan berujung KDRT kembali. Jadi Dokter D
juga perlu menyarankan untuk tetap mengkomunikasikannya dengan
suaminya terlebih dahulu.

2. Apakah dokter D memiliki kewajiban etis untuk mengungkapkan


permintaan ibu A kepada suaminya?Apakah dokter D memiliki
kewajiban untuk meminta izin kepada suami ibu A untuk memenuhi
permintaan ibu A?
→ Dokter D dapat memenuhi permintaan pasien untuk menjaga
kerahasiaan tindakan ini dari suaminya karena aspek autonomy, yaitu
pasien memiliki hak untuk memutuskan tindakan yang dilakukan
kepada tubuhnya. Tidak ada kewajiban bagi Dokter D ataupun tenaga
medis lainnya untuk mengungkapkan kepada siapapun termasuk
pihak suami karena mengedepankan prinsip autonomy pasien.
3. Apakah dokter D harus melakukan prosedur sterilisasi kepada ibu
A?
→ Iya. Namun sebelum dilakukan tindakan, Dokter D perlu
memberikan edukasi terlebih dahulu terkait sterilisasi dan memberikan
opsi penggunaan alat kontrasepsi yang lainnya setelah pasien tetap
ingin melakukan prosedur sterilisasi.

4. Buatlah analisis berdasarkan wawasan bioetika dan humaniora!


→ Pasien memiliki kebebasan menentukan tindakan yang
dilakukan pada dirinya sendiri dan mendapatkan haknya untuk
dirahasiakan dari suaminya. Persetujuan tindakan kedokteran
diberikan oleh pasien atau keluarga terdekat setelah
mendapatkan penjelasan dokter mengenai tindakan yang akan
dilakukan. Seorang dokter juga perlu menghargai prinsip
autonomy dari pasien dengan tidak memberitahukan ke
suaminya. Dalam prinsip beneficence, tindakan yang dilakukan
terhadap pasien lebih menguntungkan dari segi kesehatan dan
sosioekonominya dibandingkan kerugiannya. Dalam prinsip
non-maleficence, kita harus melindungi dan tidak merugikan
pasien.

Dalam kasus ini, kita perlu menjelaskan secara lengkap


mengenai tindakan sterilisasi yang dilakukan termasuk risiko
meninggalkan bekas luka yang berisiko ketahuan oleh suaminya
dan berujung ke KDRT. Sehingga, sebagai dokter kita tetap
menyarankan terlebih dahulu kepada pasien untuk
mengkomunikasikannya kepada suaminya sebelum dilakukan
tindakan. Saran untuk berdiskusi dengan suaminya ini juga
dikarenakan pertentangan terhadap budaya lokal. Dalam aspek
humaniora/budaya, sepasang suami-istri seharusnya memiliki hak
dan kewajiban yang sama untuk menentukan kelahiran. Namun,
tetap perlu memperhatikan juga kondisi kesehatan dan
sosioekonomi dari pasien. Mungkin dari Dokter D dapat
menyarankan untuk berdiskusi dengan suaminya dan
memberitahukan mengenai risiko medis dan ekonomi jika tidak
disterilisasi.
Case 2
● Seorang wanita berusia 24 tahun datang ke unit gawat darurat dengan
perdarahan pervaginam berat bersama suami dan tiga anaknya. Ini
merupakan kehamilan ke-4. Dia menderita kanker serviks stadium II-B,
usia kehamilan saat ini delapan minggu.
● Rekomendasi tindakan medis saat ini adalah melakukan embolisasi
arteri uterin atau memberikan terapi radiasi dosis tinggi untuk
menghentikan perdarahannya. Kedua pilihan tindakan ini
menyebabkan wanita ini kehilangan janinnya.
● Wanita ini hanya dapat berbicara dengan bahasa asing, setelah dia
bicara dengan suaminya, suaminya menjelaskan kepada dokter
bahwa istrinya menolak semua tindakan yang akan menghilangkan
janinnya.
● Beberapa menit kemudian, perdarahan wanita tersebut semakin
parah, mulai terjadi penurunan kesadaran, keadaan umum semakin
memburuk, dan dokter mengatakan kepada suaminya bahwa jiwa istri
dan calon anaknya saat ini sedang terancam.
● Suaminya mengatakan kepada dokter untuk melakukan tindakan
apapun yang bisa menyelamatkan nyawa istrinya.

1. Dilema etik apa yang dihadapi oleh tim perawatan kesehatan?


o Pasien sedang mengandung usia 8minggu → ibu hamil merupakan
pengecualian dalam etika medis dengan prinsip utama Primum non
nocereàakses kepada fetus hanya dpt mll intervensi pada ibu hamil
dan mengobati ibu dapat membahayakan janin
oPasien yang sedang mengandung hamil dan mengidap kanker servik
stadium IIB → heavy bleeding → dokter menyarankan utk mengobati
namun dpt menyebabkan kematian pd janinàibu menolak

o Pengobatan antineoplastic dpt mempengaruhi janin yg sedang


berkembang à janin belum memiliki autonomous choice

2. Hal apa yang mungkin dipertimbangkan oleh tim


perawatan kesehatan? Hal apa yang bisa menjadi
alasan wajar untuk menerima penolakan pasien

o Dokter Harus Mengerti Mindset,broad social network,value,kebudayaan,


kepercayaan (agama) yg mempengaruhi keputusan ibu hamil
o Jelaskan mengenai treatment plan dan cancer outcome kepada ibu →
meminta informed consent → keputusan ibu absolute dan tidak terbatas
o Jelaskan Mengenai Self Benefit(membantu kondisi ibu),benefir untuk anak
(minimalisir resiko pd fetus), dan altruism (partisipasi dpt membantu Wanita
dan pengembangan ilmu)

o Dapat Meminta Second Opinion

3. Apa yang harus dilakukan oleh penyelia layanan


kesehatan?
o Pertimbangkan autonomy pasien dg cara informed consent,pastikan
pasien mengetahui mengenai fakta-fakta medis, seperti progonosis cancer,
kemungkinan terapi antineoplastic, toksisitas dan tujuan, meningkatkan QOL,
overall and progression0free survival

o Pertimbangkan timing: Ibu symptomatic → apakah harus di treat segera


atau bisa ditunda hingga trimester 3

o Pertimbangkanbeneficencedannonmaleficence

4. Berikan pandangan berdasarkan wawasan bioetika dan


humaniora
Case 3
● Dokter D adalah satu-satunya dokter yang melayani
komunitas kecil di pinggiran kota. Dokter D sedang
merawat JT, laki-laki berusia 22 tahun putra dari
keluarga T untuk infeksi kelamin aktif yang menurut JT
mungkin tertular dari penari eksotis yang bekerja di
kub malam di kota terdekat. Pada pengujian JT, dokter D
tidak menemukan bukti bahwa ia telah tertular HIV.
● Ketika pasien dokter D yaitu tuan MA ada di kantor dokter
D untuk perawatan, dia memberi tahu dokter D bahwa
putrinya yang berusia 17 tahun akan menikah dengan
JT, yang diketahui tuan MA sebagai pasien dokter D.
● Tuan MA bertanya kepada dokter D mengenai status
Kesehatan JT. Dia menghawatirkan JT membawa
sumber penyakit yang bisa menyebabkan penderitaan
kepada anaknya. Dia mengatakan informasi kesehatan
JT sangat penting untuk kelangsungan kesejahteraan
hidup anaknya.

1. Apa dilemma etik dalam kasus ini?


Pasien memiliki hak untuk dijaga kerahasiaannya dan
tidak boleh disebarluaskan kepada siapapun. Didukung aspek
justice, keadilan memperlakukan seluruh pasien dengan
sama. Pada kasus ini, seorang dokter berkewajiban untuk
menjaga kerahasiaan pasien. Oleh sebab itu, tidak dapat
memberitahukan informasi personal pasien kepada siapapun
itu tanpa izin kepada pasien tersebut.

2. Apa yang mungkin disampaikan oleh dokter D?


Untuk menjaga profesionalitas dan tidak menyinggung Tuan
MA, Dokter harus bisa memberikan pengertian secara jelas
bahwa sebagai seorang dokter tidak diperbolehkan untuk
memberikan data pribadi seorang pasien, sama halnya
Dokter D yang tidak memberikan data pribadi Tuan MA
kepada orang lain. Dokter D mungkin dapat memberikan
saran kepada putri Tuan MA untuk melakukan tes kesehatan
bersama siapapun calon pasangannya nanti. Cara
penyampaiannya:
“Tuan MA, APA YA INI DIBIKIN JG YA???” BEBASSSS

Berdasarkan prinsip bioetika Confidentiality, Dr.DL harus tetap menjaga


kerahasiaan status kesehatan JT kepada orang lain termasuk pada calon
istrinya. Hal ini dilakukan salah satunya untuk mencegah timbulnya rasa
takut pasien penderita IMS untuk memeriksakan diri ke fasilitas
kesehatan. Dr. DL harus menghubungi JT untuk menanayakan apakah
JT sudah memberi tahu istrinya terkait penyakit serta perawatan yang
sedang ia jalani. Selain itu, Dr.DL juga perlu menyarankan Mr.MA agar
putrinya bisa berdiskusi dan saling terbuka terkait status kesehatannya
dengan JT dan juga menyarankan agar putrinya dan JT melakukan
pre-marital screening terlebih dahulu sebelum pernikahan.

3. Dengan siapa dokter D dapat berkonsultasi dalam kasus


ini?
Inii konsultasi sama siapa ya… ke jt kali ya? Kyk jelasin lu harus
sampein medical info lu ke calon istrilu gitu”? Ohh oke thank
you
The HIPAA Privacy Rule at 45 CFR 164.510(b) specifically permits
covered entities to share information that is directly relevant to the
involvement of a spouse, family members, friends, or other persons
identified by a patient, in the patient’s care or payment for health care.

Berdasarkan permenkes nomor 36 tahun 2012 pasal 5, rahasia


kedokteran dapat dibuka untuk kepentingan kesehatan pasien, memenuhi
permintaan aparatur penegak hukum dalam rangka penegakan hukum,
permintaan pasien sendiri, atau berdasarkan peraturan
perundang-undangan

Dr. DL dapat berkonsultasi mengenai informasi kesehatan pasien kepada


anggota keluarga yang sah secara hukum, istri, teman, atau orang yang
bertanggung jawab atas perawatan dan biaya kesehatan pasien, jika
pasien tidak keberatan.

Oleh karena itu, Dr. DL dapat menghubungi JT terlebih dahulu untuk


mengingatkannya tentang tanggung jawabnya terhadap pasangan seksual,
dan untuk menanyakan apakah dia telah memberi tahu putri Tn. MA
tentang kesehatannya.

Apabila Dr. D ingin menyampaikan informasi, Dr. D harus berkonsultasi dan


mengonfirmasi pada JT terlebih dahulu apakah JT sudah memberikan
informasi kepada calon istrinya terkait perawatan dan tes kesehatan yang
ia jalanin. Dr. D juga dapat menyampaikan pada JT untuk terbuka dan jujur
serta melakukan screening bersama sebelum menikah.

09.45 disubmitt
4. Buatlah analisis berdasarkan wawasan bioetika dan
humaniora! Ini autonomy beneficence non maleficence sama
justice ditulis semua? Iyaaa, di pdf ada ga? Kaga ada
Justice: menjaga kerahasiaan pasien seperti dokter. D
memberikan perlakuan kepada seluruh pasien. Tidak
membeda-bedakan dengan kasus apapun dan harus
berkonsultasi terkait kondisi kesehatan dengan
pasien/keluarga/orang yang merawat pasien.

Autonomy: dokter D harus mampu berpikir logis dan mampu


mengambil keputusannya sendiri. Disini dokter D dituntut
untuk mengkritisi dan tidak menyebarluaskan informasi
pasien dalam bentuk apapun kepada siapapun sebelum
mendapatkan persetujuan pasien.

Beneficence: Karena sudah diatur dalam aspek hukum, dr. D


diharapkan bisa mengikuti regulasi dan melakukan
pencegahan dari kesalahan seperti memberikan kondisi
kesehatan pasien. Dokter D harus mampu memilih perbuatan
yang baik bagi JT dan Tuan MA. Selain itu, dokter D juga harus
menjawab pertanyaan tuan MA sesopan dan sebaik mungkin.

Nonmaleficence: disini sangat penting menjaga kerahasiaan


status kesehatan JT karena dapat berimplikasi pada kondisi
mental JT apabila ternyata mengalami HIV dan juga
disebarluaskan informasinya. JT akan merasa malu dan
dapat mengalami kondisi psikologis karena ketidaksiapan
akan penerimaan dirinya, oleh sebab itu harus memiliki posisi
yang tidak merugikan pasien.

CASE 3 DARI BUKU


Mungkin secara etis sulit bagi Dr. DL untuk melakukan selain
mengingatkan
Pak MA bahwa seorang dokter harus menjaga rahasia pasien
informasi, dengan cara yang sama bahwa Dr. DL tidak dapat
mendiskusikan Mr. MA
status kesehatan dengan tetangga. Ini akan menjadi
masalah bagi Dr.
DL bahkan mengatakan bahwa JT tidak membawa HIV, virus
AIDS, karena
ini dapat diartikan bahwa ia tidak memiliki infeksi menular.
Lebih lanjut, Tuan MA mungkin mengetahui bahwa, meskipun
ada cara lain untuk
penularan, HIV adalah infeksi menular seksual (IMS), dan
menyadari bahwa, jika Dr. DL sedang menguji JT untuk IMS,
dan hanya menyatakan satu
bahwa dia tidak memiliki, dia mungkin memiliki yang lain.
Demikian,
Dr DL mungkin harus menasihati Mr MA bahwa putrinya,
seperti apapun
calon pengantin, harus meminta JT agar mereka bertukar
kesehatan
informasi tentang diri mereka sendiri.
Namun, dalam keadaan luar biasa, Dr. DL dapat dibenarkan
atau dimaafkan secara etis dalam melanggar kerahasiaan
JT, tetapi tidak
tentu kepada Pak MA. Jika putrinya meminta Dr. DL untuk
resep kontrasepsi, menjelaskan bahwa dia dan JT adalah,
atau
akan menjadi, aktif secara seksual satu sama lain, Dr. DL
mungkin bertindak membela diri dalam menanyakan
padanya apakah JT telah membahasnya
status medis. Apakah tingkat pengungkapan ini oleh Dr.
DL memenuhi standar etika, pengadilan telah menyatakan
bahwa a
perlindungan dokter dari orang yang rentan dengan biaya a
kerahasiaan pasien dapat dimaafkan secara hukum, dan
tidak
menimbulkan tanggung jawab hukum.
Oleh karena itu, sementara Dr. DL mungkin lebih baik
disarankan untuk membalas dengan
menjaga kerahasiaan medis JT, terutama karena takut
bahwa
orang akan menghindari mencari perawatan medis jika
kerahasiaannya
dilanggar, mungkin ada keadaan luar biasa yang signifikan
bahaya bagi orang yang rentan yang tidak dapat dicegah
dengan cara lain
cara yang secara etis membenarkan atau memaafkan
pengungkapan, asalkan
bahwa ruang lingkup pengungkapan adalah yang paling
sedikit yang akan melayani
tujuan pencegahan. Dr DL dapat menghubungi JT untuk
mengingatkannya tentang
tanggung jawabnya terhadap pasangan seksual, dan untuk
menanyakan apakah
dia telah memberi tahu putri Tuan MA tentang perawatannya
Case 4
● Dokter RR adalah dokter yang merawat seorang ibu
hamil yang bernama nyonya TP. Nyonya TP hamil anak
pertama setelah sebelas tahun penantian dalam
pernikahannya.
● Pada akhir trimester ketiga, dokter RR mendiagnosis
nyonya TP mengalami komplikasi di akhir kehamilannya
yaitu placenta praevia totalis.
● Nyonya TP berniat menjalani proses kelahiran secara
normal, hal ini dia yakini karena melahirkan dengan cara
normal merupakan anugerah dari ilahi.
● Dokter RR menyimpulkan bahwa keinginan nyonya TP
akan membahayakan nyawanya serta nyawa janin yang
ada dalam kandungannya, bahkan dapat menyebabkan
kematian bagi keduanya.
● Nyonya TP yang sangat taat terhadap agamanya, yakin
bahwa dia dan bayinya akan terlahir dalam keadaan
selamat dan sehat, dalam keyakinannya tersebut bahwa
semua permasalahan akan selesai dengan intervensi
ilahi.
● Secara medis, tindakan persalinan yang paling mungkin
untuk menyelamatkan kedua nyawa ibu dan janin
adalah dengan melaksanakan operasi sesar.

1. Dilema etik apa saja yang mungkin akan muncul?


Sebagai dokter dilema etik yang muncul berkaitan dengan dua prinsip bioetik yaitu
prinsip autonomy dan prinsip non-maleficence. Prinsip autonomy menjelaskan
bahwa kita harus menghormati keputusan pasien untuk memilih treatment dan kita
tidak boleh memaksakan treatment terhadap pasien oleh karena itu kita harus
menghormati keputusan nyonya TP untuk menjalani proses persalinan normal.
Namun, sebagai seorang dokter, kita juga tidak boleh melakukan tindakan yang
akan mencelakai pasien. Larangan tersebut ialah dasar dari prinsip bioetik
non-maleficence, sehingga kita juga seharusnya tidak boleh melakukan proses
persalinan normal dalam kasus ini karena dapat membahayakan nyawa, baik
nyawa nyonya tp dan bayinya.

2. Dapatkah dokter RR memaksakan persalinan sesar atas nyonya TP?

3. Dapatkah dokter RR meminta pengadilan untuk mengizinkan melaksanakan


operasi sesar terhadap nyonya TP?

4. Buatlah analisis berdasarkan wawasan bioetika dan humaniora!


Sebagai dokter RR, hal yang pertama yang dapat kita lakukan adalah menjelaskan
kepada nyonta TP dan keluarganya mengenai komplikasi kehamilan yang muncul.
Penjelasan tersebut harus menggunakan bahasa yang mudah dimengerti dan
harus juga dijelaskan secara penuh dan jelas tanpa ada hal yang ditambahkan dan
dikurangi. Kemudian, penjelasan juga harus meliputi tentang adanya risiko kematian
pada anak dan juga ancaman terhadap nyawa nyonya TP apabila kelahiran
dilakukan secara normal. Penjelasan mengenai ancaman kepada nyawa harus kita
tekankan akibat nyonya TP yang memilih untuk melahirkan secara normal. Apabila
nyonya TP tetap memilih untuk melahirkan secara normal maka kita harus
menjelaskan bahwa dalam proses melahirkan nanti apabila memang betul-betul
tidak memungkinkan dan janin memang sudah akan mati, maka operasi sesar
emergency harus dilakukan untuk menyalamatkan nyawa nyonya TP. Kemudian, kita
harus memastikan bahwa nyonya TP dan keluarga benar-benar sudah jelas dengan
informasi yang sudah dijelaskan, apabila belum maka harus kita terus jelaskan
sampai semuanya paham dengan menyeluruh. Yang terakhir, kita harus
memastikan bahwa nyonya TP dan keluarga menandatangani consent form yang
menyatakan bahwa keputusan mereka tidak ada paksaan dan dari pihak rumah
sakit sudah menjelaskan secara detil dan jelas dan mereka sudah paham terkait
bahaya yang akan timbul dari prosesi persalinan normal.
Pada kasus ini, dokter RR harus menerapkan prinsip bioetik autonomy dengan
menghargai keputusan nyonya TP untuk melahirkan melalui persalinan normal dan
tidak boleh memaksakan nyonya TP untuk melakukan prosesi persalinan sesar.
Selanjutnya, prinsip bioetik nonmaleficence juga bermain peran dimana apabila
saat proses persalinan janin sudah tidak dapat diselamatkan, maka nyawa nyonya
TP harus diselamatkan sehingga operasi sesar emergency harus dilakukan untuk
menyelamatkan nyawa nyonya TP.

You might also like